Vous êtes sur la page 1sur 82

1. Patients with Parkinson's disease are more likely to fall.

Which of the following is not a


common contributing factor?
A. Peak.-dose dyskinesias
B. Off periods
C. Impaired postural reflexes
D. Convulsive motor activity
(Jawab:D REVIEW MANUAL FOR NEUROLOGY IN CLINICAL PRACTICE 4TH
EDITION no 3-2)
Questions 2 through 12:
The following questions present brief vignettes. For each, select the correct diagnosis
from the following list.
E. Absence seizme
F. Complex partial seizure
G. Both
H. Neither
2. 24-year-old man with episodes ofloss of consc.iousness without loss ofposrure, followed
by several minutes ofconfusion.
(Jawab:B REVIEW MANUAL FOR NEUROLOGY IN CLINICAL PRACTICE 4TH
EDITION no 2-10)
3. Breath-holding spells are differentiated from seizures by which of the following features?
A. Breath-holding spells occur several times per day whereas seizures are usually
less common.
B. Clonic activity is seen only with epilepsy and not with breath- holding pells
C. Breath-holding spells occur without provocation, whereas epilepsy is frequently startle
induced
D. Incontinence is only seen with epilepsy and not with breath-holding spells
E. All ofthe above
(Jawab:A REVIEW MANUAL FOR NEUROLOGY IN CLINICAL PRACTICE 4TH
EDITION no 2-14)
4. 16-year-old female with episodes of loss of responsiveness for a few s.:conds without loss
of posture. Normal function immediately following the episode.
(Jawab:A REVIEW MANUAL FOR NEUROLOGY IN CLINICAL PRACTICE 4TH
EDITION no 2-11)
5. Aging predisposes to falls by which ofthe following mechanisms?
1. Impaired proprioception
2. Arthritis
3. Impaired vision
4. Loss of muscle bulk

Select: A =1,2,3. B =I, 3. C =2, 4. D =4 only. E = All


(Jawab:E REVIEW MANUAL FOR NEUROLOGY IN CLINICAL PRACTICE 4TH
EDITION no 3-3)
6. Cataplexy is an uncommon cause of falls and is associate with narcolepsy. Which of
the following is not a cardinal feature of narcolepsy?
A. Excessive daytime sleepiness
B. Hypnagogic hallucinations
C. Poor nocturnal sleep
D. Sleep paralysis
(Jawab:C REVIEW MANUAL FOR NEUROLOGY IN CLINICAL PRACTICE 4TH
EDITION no 3-4)\
7. How can cataplexy be differentiated from epilepsy?
A. Epileptic syncope does not vccur without other signs ofconvulsive activity
B. Cataplexy is provoked wh:reas epilepsy is typically not provoked
C. Consciousness is preserved in cataplexy but disturbed in patiCllts with
epilepsy
D. Arrhythmias are typical ofcataplexy but not seizures
(Jawab:B REVIEW MANUAL FOR NEUROLOGY IN CLINICAL PRACTICE 4TH
EDITION no 3-5)
Questions 8 through 11:
The following questions present brief clinical information. For each, select the condition
which is most likely to be associated with the disorder.
A. Parkinson's disease
B. Progressive Supranuclear palsy
C. Both
D. Neither
8. Associated with rigidity.
(Jawab:C REVIEW MANUAL FOR NEUROLOGY IN CLINICAL PRACTICE 4TH
EDITION no 3-6)
9. Greater tendency to fall backwards.
(Jawab:B REVIEW MANUAL FOR NEUROLOGY IN CLINICAL PRACTICE 4TH
EDITION no 3-7)
10. ssociated with spasticity.
(Jawab:B REVIEW MANUAL FOR NEUROLOGY IN CLINICAL PRACTICE 4TH
EDITION no 3-8)
11. May be preceded by REM sleep behavior disorder.
(Jawab:B REVIEW MANUAL FOR NEUROLOGY IN CLINICAL PRACTICE 4TH

EDITION no 3-9)
12. Which of the following statements concerning vestibular causes off falls are true?
1. Patients with Meniere's disease may suddenly fall without accompanying vertigo
2. Aberrant signals from vesnbular receptors result in inappropriate postural
adjustment
3. Patients with vertigo can have gait ataxia and falls without appendicular ataxia
4. Peripheral vestibulopathy is the most likely cause ofcryptogenic falls in middleaged women
Select A =1,2,3. B= 1,3. c= 2, 4. D =4 only. E= All
(Jawab:A REVIEW MANUAL FOR NEUROLOGY IN CLINICAL PRACTICE 4TH
EDITION no 3-10)
13. Drop attacks may be exacerbated by carbamazepine in patients with Rolandic
epilepsy.
T. True
F. False
(Jawab:T REVIEW MANUAL FOR NEUROLOGY IN CLINICAL PRACTICE 4TH
EDITION no 3-13)
14. Which of the following statements regarding depression in Parkinson's disease are true ?
1. Depression is a common accompaniment
2. Psychosis is common
3. Patients with major depression progress faster in PO than patients without depression
4. Depression in PD is more likely with
older age of onset of disease
Select: A; 1,2,:1. B; 1,3. C = 2,4. D =4 only. E= All
(Jawab:A REVIEW MANUAL FOR NEUROLOGY IN CLINICAL PRACTICE 4 TH
EDITION no 8-9)
Questions 15 through 17:
Behavioral alterations are common in epilepsy. For each behavioral state, select whether
the condition is more likely to be ictal, postictal, or interictal.
A. Ictal
B. Postictal
C. Interictal
D. Any of the above
15. Confusion.
(Jawab:B REVIEW MANUAL FOR NEUROLOGY IN CLINICAL PRACTICE 4 TH
EDITION no 8-13)
16. Depression.

(Jawab:D REVIEW MANUAL FOR NEUROLOGY IN CLINICAL PRACTICE 4 TH


EDITION no 8-14)
17. Unprovoked laughter.
(Jawab:A REVIEW MANUAL FOR NEUROLOGY IN CLINICAL PRACTICE 4TH
EDITION no 8-15)
18. Patients with Parkinson's disease have which features of speech?
1. Decreased and monotonous speech
2. Increased rate of speech
3. Absence of fluctuation in loudness and pitch
4. Consonant substitution
Select: A = 1.2.3. B = 1. 3 C = 2. 4. 0 = 4 only. E = All
(Jawab:E REVIEW MANUAL FOR NEUROLOGY IN CLINICAL PRACTICE 4TH
EDITION no 12B-1)
19. Drug toxicity is an important cause of vestibulopathy. Which of the following statements
are true about drug-induced vestibulopathy?
1. Streptomycin and gentamycin have greater vestibulotoxic effects than other drugs of
the class
2. Risk of aminoglycoside vestibulopathy is increased with renal failure
3. Patients have clinical findings of bilateral dysfunction
4. Patients have examination findings of vestibulopathy but seldom have neurologic
deficit
Select: A = I. 2. 3. B = 1. 3. C = 2. 4. D = 4 only. E = All
(Jawab:A REVIEW MANUAL FOR NEUROLOGY IN CLINICAL PRACTICE 4 TH
EDITION no 18-2)
20. A 40-year-old man presents with episodes of severe vertigo and vomiting, associated with
tinnitus, fluctuating hearing loss, and a pressure sensation in the ear. Which is the most
likely diagnosis?
A. Acoustic neuroma
B. Meniere's disease
C. Brainstem infarction
D. Toxic vestibulopathy
(Jawab:B REVIEW MANUAL FOR NEUROLOGY IN CLINICAL PRACTICE 4 TH
EDITION no 18-4)
21. Benign paroxysmal positional vertigo is suggested by episodic vertigo induced by certain
head positions, with a history of either labyrinthitis with a URI or head injury.
T. True
F. False
(Jawab:T REVIEW MANUAL FOR NEUROLOGY IN CLINICAL PRACTICE 4 TH
EDITION no 18-8)

22. Patients with Parkinson's disease have impaired olfactory detection.


T. True
F. False
(Jawab:T REVIEW MANUAL FOR NEUROLOGY IN CLINICAL PRACTICE 4 TH
EDITION no 20-9)
23. Which of the following statements concerning activity-dependent headache are true?
1. Most exertional headaches are benign
2. Exertional headache is worrisome for posterior fossa lesion
3. Positional headache exacerbated by upright posture suggests low CSF pressure
4. Recurrent headache during sexual activity is most likely due to unruptured aneurysm
Select:A= 1,2,3. B= I, 3.C =2.4. D= 4 only. E=AII
(Jawab:A REVIEW MANUAL FOR NEUROLOGY IN CLINICAL PRACTICE 4TH
EDITION no 21-1)
24. Which of the following are true regarding cluster headache?
1. Headache is usually bilateral but may be unilateral
2. Headache may change sides between clusters
3. Alcohol precipitates migraine but not cluster headache
4. Pain of cluster may be in the lower half of the face
Select: A = 1,2,3. B = 1,3. C = 2, 4. D = 4 only. E = All
(Jawab:C REVIEW MANUAL FOR NEUROLOGY IN CLINICAL PRACTICE 4 TH
EDITION no 21-2)
25. Migraine aura can be difficult to differentiate from brain ischemia. Which of the
following are differentiating features?
1. Aura symptoms spread over minutes whereas stroke symptoms have an abrupt onset
2. If more than one modality is affected by an aura, they have a staggered onset
3. Visual symptoms are the most common auras
4. Aura symptoms which develop during the headache phase are more likely to be due
to migrainous infarction
Select: A = 1,2, 3. B = 1,3. C = 2,4.0=4 only. E = All
(Jawab:A REVIEW MANUAL FOR NEUROLOGY IN CLINICAL PRACTICE 4 TH
EDITION no 21-3)
26. Which of the following clinical features suggests temporomandibular joint dysfunction?
1. Clenching of the teeth
2. Tenderness in the temporal region
3. TMJ clicking
4. Increased range of motion of the TMJ
Select: A = 1,2,3. B = 1,3. C = 2, 4. D = 4 only. E = All
(Jawab:B REVIEW MANUAL FOR NEUROLOGY IN CLINICAL PRACTICE 4 TH
EDITION no 21-4)
27. Some headache syndromes are hereditary. Which of the following statements regarding
hereditary are true?
1. Migraine is often inherited

2. Cluster is inherited in Iess than 10% Of cases


3. Familial hemiplegic migraine is inherited as an autosomal dominant trait
4. Tension headache has a familial tendency
Select; A = 1,2,3. B = 1,3. C = 2,4. D = 4 only. E= All
(Jawab:E REVIEW MANUAL FOR NEUROLOGY IN CLINICAL PRACTICE 4 TH
EDITION no 21-5)
Questions 21-6 through 21-10:
For the following questions, indicate the most likely diagnosis from the following list:
A. Migraine
B. Cluster headache
C. Tension headache
D. Increased intracranial pressure
E. Decreased intracranial pressure
F. Trigeminal neuralgia
G. TMJ pain
28. Headache which is markedly worsened by upright posture, and relieved by lying down.
(Jawab:E REVIEW MANUAL FOR NEUROLOGY IN CLINICAL PRACTICE 4 TH
EDITION no 21-6)
29. Episodic unilateral throbbing headache without neurologic symptoms or signs. The
patient has nausea and photophobia with the attacks, but no symptoms between attacks.
(Jawab:A REVIEW MANUAL FOR NEUROLOGY IN CLINICAL PRACTICE 4 TH
EDITION no 21-7)
30. Headache which is worse in the morning, is better when upright.
(Jawab:D REVIEW MANUAL FOR NEUROLOGY IN CLINICAL PRACTICE 4 TH
EDITION no 21-8)
31. Episodes of unilateral lancinating pain lasting seconds to a minute. No neurologic deficit.
(Jawab:F REVIEW MANUAL FOR NEUROLOGY IN CLINICAL PRACTICE 4 TH
EDITION no 21-9)
32. Episodes of severe but brief unilateral periorbital headache.
(Jawab:B REVIEW MANUAL FOR NEUROLOGY IN CLINICAL PRACTICE 4TH
EDITION no 21-10)
33. A patient presents with hemisensory loss with pain on the same side of the body. There is
no weakness. Which of the following statements are true regarding this condition?
1. Proprioception is preserved
2. The most likely diagnosis is mUltiple sclerosis
3. The pain is always stimulus-independent
4. The lesion is most likely in the entroposterolateral nucleus of the thalamus
Select: A = 1.2.3. B = 1. 3. C = 2,4 . D = 4 only. E = All

(Jawab:D REVIEW MANUAL FOR NEUROLOGY IN CLINICAL PRACTICE 4 TH


EDITION no 22-5)
34. Which of the following best described the speech difficulty in patients with Parkinson's
disease?
A. Palilalia
B. Slurred speech with decreased volume
C. Stuttering speech with defective prosody and inappropriate pauses
D. Fluent sounding speech devoid of content
(Jawab:B REVIEW MANUAL FOR NEUROLOGY IN CLINICAL PRACTICE 4 TH
EDITION no 24-5)
35. Which of the following statements about Parkinson's disease are true?
1. Observation of Keyser-Fleischer rings on the cornea confum the diagnosis
2. Handwriting becomes smaller and slower
3. Marked voice changes are common early in the course
4. Motor findings are commonly asymmetric early in the course
Select A= 1,2.3. B = 1.3. C=2.4. D = 4 only. E= All
(Jawab:C REVIEW MANUAL FOR NEUROLOGY IN CLINICAL PRACTICE 4 TH
EDITION no 24-7)
36. Which of the following causes of akinetic rigid syndrome can differentiate PSP from
Parkinson's disease?
A. Neck posture is flexed in PSP rather than extended as in PO
B. Neck posture is extended in PSP rather than flexed as in PO
C. Early loss of postural reflexes is seen in PO rather than PSP
D. Cognitive dysfunction is seen in PD but not PSP
(Jawab:B REVIEW MANUAL FOR NEUROLOGY IN CLINICAL PRACTICE 4TH
EDITION no 25-3)
37. Which of the following features suggest Parkinson's disease rather thansymptomatic
parkinsonism?
A. Narrow based gait
B. Shuffling gait
C. Freezing
D. Hesitation to initiate gait
(Jawab:A REVIEW MANUAL FOR NEUROLOGY IN CLINICAL PRACTICE 4 TH
EDITION no 25-5)
38. What is the most likely diagnostic consideration in a patient with arm pain plus weakness
of the finger and wrist flexors and interossei?
A. C8 radiculopathy from cervical spondylosis
B. Radiation plexitis
C. Mononeuropathy multiplex
D. Tumor infiltration of the brachial plexus

(Jawab:D REVIEW MANUAL FOR NEUROLOGY IN CLINICAL PRACTICE 4 TH


EDITION no 26-2)
39. Patients with myotonia often complain of muscle cramps. Which ofthe following best
describes the EMG fmdings in myotonia?
A. Repetitive bursts of muscle fiber action potentials exacerbated by mechanical
stimulation
B. Repetitive bursts of motor unit action potentials, exacerbated by mechanical
stimulation
C. Continuous motor unit action potentials which can be briefly suppressed by
voluntary effort
D. Continuous motor unit action potentials which can only be blocked by local
anesthetic injection
(Jawab:A REVIEW MANUAL FOR NEUROLOGY IN CLINICAL PRACTICE 4 TH
EDITION no 29-3)
40. Which of the following disorders is most likely to produce muscle pain?
A. Becker's muscular dystrophy
B. Lovastatin myopathy
C. Mitochondrial myopathy
D. Duchenne's muscular dystrophy
E. Polymyositis
(Jawab:B REVIEW MANUAL FOR NEUROLOGY IN CLINICAL PRACTICE 4 TH
EDITION no 29-4)
41. Which of the following disorders which cause muscle cramps are characterized by
electrical silence on EMG?
1. Brody's syndrome
2. Hypothyroidism
3. Myophosphorylase deficiency
4. Stiff-man syndrome
Select: A= 1.2.3. B = 1,3. C =2.4. D=4 only. E= All
(Jawab:A REVIEW MANUAL FOR NEUROLOGY IN CLINICAL PRACTICE 4 TH
EDITION no 29-5)
42. A 76-year-old woman presents with pain in the right hip and proximal thigh and
difficulty arising from a chair. Which is the most likely diagnosis?
A. Polymyositis
B. Amyotrophic lateral sclerosis
C. Myasthenia gravis
D. Degenerative disease of the hip
(Jawab:D REVIEW MANUAL FOR NEUROLOGY IN CLINICAL PRACTICE 4 TH
EDITION no 29-6)
43. Which of the following would not be an appropriate treatment for muscle cramps?

A. Quinine'
B. Carbamazepine
C. Phenytoin
D. Albuterol
(Jawab:D REVIEW MANUAL FOR NEUROLOGY IN CLINICAL PRACTICE 4 TH
EDITION no 29-7)
44. Which of the following statements is true regarding neurogenic muscle cramps?
A. The cramps are due to electrical activity in the peripheral nerves but the muscle
fibers are electrically silent
B. Neurogenic muscle cramps always have their origin in the peripheral nerves
C. CNS lesions can produce muscle cramps
D. EMG in neurogenic muscle cramps shows repetitive discharge of individual
muscle fibers
(Jawab:C REVIEW MANUAL FOR NEUROLOGY IN CLINICAL PRACTICE 4 TH
EDITION no 29-8)
45. Which of the following is not true of chronic fatigue syndrome?
A. Patients report chronic muscle pain and fatigue
B. Sensory symptoms are absent
C. Central symptoms include headache and dizziness
D. Muscle biopsy shows non-inflammatory muscle fiber degeneration
(Jawab:B REVIEW MANUAL FOR NEUROLOGY IN CLINICAL PRACTICE 4 TH
EDITION no 29-9)
46. A 23-year-old man presents with pain and tenderness in the quadriceps. He climbed a
small mountain several days previously, but the pain did not develop until the next day.
Which is the most likely explanation?
A.Subclinical muscular dystrophy exacerbated by the exercise
B. Rhabdomyolysis from the untrained exertion
C. Delayed onset muscle soreness
D. Psychogenic pain due to the effort
(Jawab:C REVIEW MANUAL FOR NEUROLOGY IN CLINICAL PRACTICE 4TH
EDITION no 29-10)
47. Which of the following are true regarding sensory function in trigeminal neuralgia'!
A. Sensory loss is confined to one trigeminal branch distribution
B. Sensory loss is episodic and not confined to the trigeminal distribution
C. Sensory loss can extend beyond the distnbution of the lancinating pain
D. Sensory loss is not expected, and when present suggests an alternative diagnosis
(Jawab:D REVIEW MANUAL FOR NEUROLOGY IN CLINICAL PRACTICE 4 TH
EDITION no 31-12)
48. Sexual dysfunction is common in patients with epilepsy, especially in patients with
complex partial seizures. Which of the following sexual dysfunctions are seen with
increased frequency in patients with epilepsy?

1.
2.
3.
4.

Hypersexuality
Decreased sexual interest
Erectile dysfunction
Fetishism
Select: A = 1.2.3. B = t. 3. C = 2,4. D - 4 ooly. E = All
(Jawab:E REVIEW MANUAL FOR NEUROLOGY IN CLINICAL PRACTICE 4 TH
EDITION no 32-3)

49. Bladder and bowel dysfunction is common in neurodegenerative diseases, including


Parkinson's disease. Select the incorrect statement regarding Parkinsonism.
A. Bladder dysfunction in PD usually is in the setting of moderate to advanced disease
B. The most common urodynamic abnormality is detrusor hyperreflexia
C. Bladder dysfunction in PO is more common than in MSA
D. Urinary frequency is common
(Jawab:C REVIEW MANUAL FOR NEUROLOGY IN CLINICAL PRACTICE 4 TH
EDITION no 32-4)
50. Bladder dysfunction in tbe absence of other clinical findings is likely to be a findings in
which of the following conditions?
A. MS
B. MSA
C. Diatxtic nauopathy
D. All of the above
(Jawab:B REVIEW MANUAL FOR NEUROLOGY IN CLINICAL PRACTICE 4 TH
EDITION no 32-5)
51. A patient presents with pain in the right arm which extends from below the elbow to the
last two digits of the band. The pain is steady with dysesthesias. Which is the most likely
localization of the lesion?
A. Median nerve
B. Ulnar nerve
C. Radial nerve
D. Brachial plexus
(Jawab:B REVIEW MANUAL FOR NEUROLOGY IN CLINICAL PRACTICE 4 TH
EDITION no 33-1)
52. A patient presents with numbness in the right hand which wakes the patient from sleep.
The sensory change is on the lateral aspect of the palm and the index and middle fUlgers.
Which is the most likely location of the lesion?
A. Median nerve at the wrist
B. Median nerve at the elbow
C. Ulnar nerve at the wrist
D. Ulnar nerve at the elbow
(Jawab:A REVIEW MANUAL FOR NEUROLOGY IN CLINICAL PRACTICE 4 TH
EDITION no 33-2)

53. A 54-year-old man presents with pain in the right arm which is centered near the shoulder.
There is some radiation into the upper arm but not into the lower arm or hand. There is no
motor or reflex abnormality. Manipulation of the shoulder reproduces and exacerbates the
pain. Which is the most likely diagnosis?
A. Brachial plexus injury
B. Musculocutaneous nerve injury
C. Axillary nerve injury
D. Primary shoulder pathology
(Jawab:D REVIEW MANUAL FOR NEUROLOGY IN CLINICAL PRACTICE 4 TH
EDITION no 33-3)
54. Which of the following are features of carpal tunnel syndrome?
1. Predisposition by diabetes and other causes of neuropathy
2. Loss of sensation on the thenar eminence
3. Atrophy and weakness of the abductor pollicis brevis
4. Denervation is seen in the first dorsal interosseus
Select: A = 1,2,3. B = 1, 3. C = 2, 4. D = 4 only. E = All
(Jawab: B REVIEW MANUAL FOR NEUROLOGY IN CLINICAL PRACTICE
4TH EDITION no 33-6)
55. Lesion of the posterior interosseus nerve would produce which of the following?
1. Denervation of the finger and thumb
2. extensors denervation of the extensor carpi radialis longus
3. Denervation of the extensor carpi radialis brevis
4. Denervation of the triceps
Select: A = 1,2,3. B = 1, 3. C = 2, 4. D =4 only. E = All
(Jawab:B REVIEW MANUAL FOR NEUROLOGY IN CLINICAL PRACTICE 4 TH
EDITION no 33-7)
56. Which of the following muscles is supplied by the superficial peroneal nerve?
A. Tibialis anterior
B. Vastus medialis
C. Medial gastrocnemius
D. Soleus
E. None of these
Jawab:E REVIEW MANUAL FOR NEUROLOGY IN CLINICAL PRACTICE 4 TH
EDITION no 34-1)
57. You are asked to consult on an 80-year-old man with right leg weakness. He is status-post
partial colon resection and has had a slow recovery, with bed rest for more than a week
with little ambulation. The weakness developed three days ago. Examination shows
weakness of the right tibialis anterior but other muscles are of normal strength. Reflexes
are slightly depressed at the ankle but otherwise normal. Which is the most likely
diagnosis?
A. Peroneal neuropathy
B. Lumbar radiculopathy
C. Retroperitoneal hematoma

D. Retroperitoneal abscess
Jawab:A REVIEW MANUAL FOR NEUROLOGY IN CLINICAL PRACTICE 4TH
EDITION no 34-11)
Questions 58 through 60:
The following questions concern patients with dementia. For each, select the most likely
diagnosis from the following list.
A. Alzheimer's disease
B. Hydrocephalus
C. Creutzfeldt jakob disease
D. Vascular dementia

58.
(Jawab:C REVIEW MANUAL FOR NEUROLOGY IN CLINICAL PRACTICE 4 TH
EDITION no 36A-8)
59. Normal EEG background with 8.5 Hz posterior dominant rhythm.
(Jawab:A REVIEW MANUAL FOR NEUROLOGY IN CLINICAL PRACTICE 4 TH
EDITION no 36A-9)
60. Focal slowing is seen in the right occipital region with loss of the posterior dominant
alpha. Also, there is slowing in the right temporal and bilateral frontal regions.
(Jawab:D REVIEW MANUAL FOR NEUROLOGY IN CLINICAL PRACTICE 4 TH
EDITION no 36A-10)
61. The Manin-Gruber anastomosis is an important anomaly in the innervation of muscles of
the hand which affects nerve conduction study results. Which of the following
summarizes the anastomosis findings when testing ulnar nerve conduction?
A. Sensory nerve action potential amplitude is greater with distal than proximal
stimulation
B. Sensory nerve action potential amplitude is greater with proximal than distal
stimulation
C. Motor conduction measuring abductor digiti minimi CMAP shows greater
amplitude with distal stimulation than proximal stimulation
D. Motor conduction measuring abductor digiti minimi CMAP shows greater
amplitude with proximal stimulation than distal stimulation
(Jawab:C REVIEW MANUAL FOR NEUROLOGY IN CLINICAL PRACTICE 4TH
EDITION no 36B-1)

62. What role does PET play in evaluation of dementia?


A. PET has no role in diagnosis of dementia
B. PET shows decreased neuronal activity consistent with dementia, but cannot help
to differentiate causes
C. PET can differentiate Alzheimer's disease from other causes of dementia in 90%
of patients
D. PET is a good alternative to MRI and CT for imaging patients with dementia
(Jawab:C REVIEW MANUAL FOR NEUROLOGY IN CLINICAL PRACTICE 4TH
EDITION no 37E-1)
63. Which is the effect of seizures on PET scan?
A. PET shows increased metabolism of seizure foci during the interictal and ictal
periods
B. PET shows decreased metabolism of seizure foci during the interictal and ictal
periods
C. PET does not aid in the identification of characterization of seizure foci
D. PET shows increased metabolism during a seizure and decreased metabolism in
the interictal period
(Jawab:D REVIEW MANUAL FOR NEUROLOGY IN CLINICAL PRACTICE 4TH
EDITION no 37E-2)
64. How can PET be helpful for evaluation of patients with brain tumors?
1. PET characteristics can help determine the aggressiveness of tumor
2. PET can help to differentiate radiation necrosis from tumor recurrence
3. PET can follow the therapeutic response of the tumor
4. PET can differentiate types of benign neoplasms
Select : A = 1.2.3. B = 1.3.C = 2. 4. D = 4 only. E = All
(Jawab:A REVIEW MANUAL FOR NEUROLOGY IN CLINICAL PRACTICE 4TH
EDITION no 37E-3)
65. The figure shows FDG-PET of a patient, which shows globally reduced metabolism
except for the somatosensory cortex.

Which is the most likely diagnosis?


A. Parkinson's disease
B. Vascular dementia
C. Alzheimer's disease
D. Dementia with Lewy bodies
E. Frontotemporal dementia
(Jawab:C REVIEW MANUAL FOR NEUROLOGY IN CLINICAL PRACTICE 4TH
EDITION no 37E-4)

66. The figure shows FOG-PET of a patient with a history of malignant tumor in the left
posterior temporal region status post radiation therapy. What conclusion can be made on
the basis of the data?

A. Radiation necrosis
B. Recurrent tumor
C. Infarction in the region of the previous tumor
D. Study is not conclusive
(Jawab:B REVIEW MANUAL FOR NEUROLOGY IN CLINICAL PRACTICE 4TH
EDITION no 37E-5)
67. Which of the following statements are true regarding mild cognitive impairment?
1. MCI is a normal accompaniment to aging
2. MCI cannot be arrested with current medications
3. All patients with MCI progress to AD
4. Impairment in cognitive performance other than memory predicts decline in patients with
MCI
Select: A = 1.2, 3. B = I, 3. C = 2. 4. D = 4 only. E = All
(Jawab:C REVIEW MANUAL FOR NEUROLOGY IN CLINICAL PRACTICE 4 TH
EDITION no 38-1)
68. Which of the following statements is true regarding neuropsychological changes in
Parkinson's disease?
A. Cognitive deficits develop in 30% of Patient with PD
B. Dementia develops in the majority of patients with PD
C. Cognitive deficits are present in the majority of patients with PD
D. Cholinesterase inhibitors have no effect on the treatment of AD
(Jawab:D REVIEW MANUAL FOR NEUROLOGY IN CLINICAL PRACTICE 4 TH
EDITION no 38-1)
69. What effect does epilepsy have on mortality rates?
A. No change in mortality
B. Higher mortality mainly due to motor vehicle accidents
C. Higher mortality due to multiple causes
D. Lower mortality since they have ongoing medical care
(Jawab:C REVIEW MANUAL FOR NEUROLOGY IN CLINICAL PRACTICE 4 TH
EDITION no 43-4)
70. Which is the most common cause of epilepsy in the world?
A. Trauma

B. Genetic inheritance
C. Benign tumor
D. Neurocysticercosis
(Jawab:D REVIEW MANUAL FOR NEUROLOGY IN CLINICAL PRACTICE 4 TH
EDITION no 43-5)
71. Which of the following statements are true regarding epidemiology of epilepsy?
1. Seizures are more likely at the extremes of age, in infancy and with advanced age
2. Myoclonic seizures are most common in infancy
3. Absence seizures do not begin after young adult life
4. Simple partial seizures peak in middle age
Select: A = 1.2.3. B = 1.3. C = 2. 4. D = 4 only. E = All
(Jawab:E REVIEW MANUAL FOR NEUROLOGY IN CLINICAL PRACTICE 4 TH
EDITION no 43-9)
72. What is the role of rehabilitation and physical activity for patients with
Parkinson'sdisease?
A. No benefit has been demonstrated
B. Self-guided exercise is encouraged, but formal rehabilitative training is not of
benefit
C. Rehab and exercise produces benefits which outlast the duration of the activity
D. Rehab and exercise produce benefits which are lost without continued activity
(Jawab:D REVIEW MANUAL FOR NEUROLOGY IN CLINICAL PRACTICE 4TH
EDITION no 54-10)
73. Which of the following types of peripheral nerves are most sensitive to traumatic injury?
A. Large-diameter myelinated axons
B. Small-diameter myelinated axons
C. Unmyelinated axons
D. There is no substantial difference
(Jawab:A REVIEW MANUAL FOR NEUROLOGY IN CLINICAL PRACTICE 4 TH
EDITION no 56D-1)
74. Segmental demyelination of a peripheral nerve suggests which type of injury?
A. Compression
B. Penetrating injury
C. Ischemic damage
D. Toxic damage
(Jawab:A REVIEW MANUAL FOR NEUROLOGY IN CLINICAL PRACTICE 4 TH
EDITION no 56D-2)
75. An infant presents to you with Erb's palsy with damage to the upper brachial plexus. How
would you advise the parents of the injury and prognosis?
A. The damage is from stretch and damage to the myelin sheath, and prognosis is
excellent
B. The damage is mainly to the axons from stretch, and prognosis is variable and too
soon to predict

C. The damage is to both the axons and myelin sheath, and prognosis for recovery is
poor
(Jawab:B REVIEW MANUAL FOR NEUROLOGY IN CLINICAL PRACTICE 4 TH
EDITION no 56D-3)
76. A heroin addict presents with left median neuropathy secondary to repeated injection into
the fore&rm. He never bad an injection directly into the nerve that he remembers, but has
had gradual progressive loss of left median nerve function. Examination shows the
proximal forearm near the antecubital fossa to be fibrotic. Which is appropriate
treatment?
A. Analgesics
B. Corticosteroids
C. Surgical intervention in the foreann
D. Surgery at the carpal tunnel
(Jawab:C REVIEW MANUAL FOR NEUROLOGY IN CLINICAL PRACTICE 4TH
EDITION no 56D-4)
77. A child with severe epilepsy is placed on a ketogenic diet which results in some
improvement of control of the seizures. She subsequently develops visual loss and is
found to have optic neuropathy. What is the most likely cause?
A. Mass lesion causing seizures and optic nerve compression
B. Vitamin A deficiency
C. Thiamine deficiency
D. Nicotinic acid deficiency
(Jawab:C REVIEW MANUAL FOR NEUROLOGY IN CLINICAL PRACTICE 4TH
EDITION no 63-3)
78. Which of the following suggest striatonigral degeneration rather than Parkinson's
disease?
1. Absence of resting tremor
2. Asymmetric motor signs
3. Severe autonomic dysfunction
4. Slower functional decline
Select: A = 2, 2, 3. B = I. 3. C = 2.4. D = 4 only. E = All
(Jawab:B REVIEW MANUAL FOR NEUROLOGY IN CLINICAL PRACTICE 4TH
EDITION no 72-9)
79. A patient presents with episodes of unresponsiveness without loss of postural tone. He
appears unaware of external stimuli during the episode and briefly confused after the
episode. He has no memory for events during the episode. Which is the most likely
diagnosis?
A. Attention deficit disorder
B. Absence seizure
C. Simple partial seizure

D. Complex partial seizure


(Jawab:D REVIEW MANUAL FOR NEUROLOGY IN CLINICAL PRACTICE 4TH
EDITION no 73-1)
80. Which of the following statements are true regarding mortality from epilepsy?
1. Mortality in patients with epilepsy is increased at least twofold
2. Motor vehicle accident is the most common cause of accidental death due to epilepsy
3. The cause of death in epilepsy is often related to the underlying cause
4. Sudden unexplained death (SUDEP) is unrelated to seizure control
Select: A= 1,2,3. B-1, 3. C = 2,4. D=4 only. E=All
(Jawab:B REVIEW MANUAL FOR NEUROLOGY IN CLINICAL PRACTICE 4TH
EDITION no 73-3)
81. Which of the following statements are true regarding post-traumatic epilepsy?
1. Post-traumatic epilepsy is most common in older patients
2. Post-traumatic epilepsy is increased in incidence in patients with intracranial hematoma
3. Prophylaxis with phenytoin lowers the risk of late development seizures
4. Post-traumatic epilepsy is more common with depressed skull fractures
Select: A = 1, 2, 3. B = 1,3. C = 2. 4. D = 4 only. E = All
(Jawab:C REVIEW MANUAL FOR NEUROLOGY IN CLINICAL PRACTICE 4TH
EDITION no 73-4)
82. A 36-year-old female presents withepisodes of unresponsiveness associated with flailing
of the arms and pelvic thrusting. The head swings from side to side. The patient is limp
and confused after the episode for a few minutes. Which is the most appropriate
formulation?
A. The episodes are psychogenic seizures and psychiatric evaluation is indicated
B. The episodes are probably psychogenic, but monitoring should be perfomedto
confirm the diagnosis
C. The episodes are complex partial seizures and AEO therapy is needed
D. The episodes are generalized seizures and AEO therapy is needed
(Jawab:B REVIEW MANUAL FOR NEUROLOGY IN CLINICAL PRACTICE 4TH
EDITION no 73-5)
83. Which of the following statements are true regarding evaluation of patients with
potential non-epileptic seizures?
1. Normal EEG during a partial or generalized seizure is diagnostic ofnon epilepric seizure
2. Presel'yed posterior dominant rhythm during seizure is diagnostic of non-epileptic
seizure
3. Lack of elevation in serum prolactin with a generalized seizure makes the diagnosis of
nonepileptic seizure
4. Suppression ofEEG background after a clinical seizure indicates an epileptic seizure
Select: A= 1, 2, 3. B = 1,3. C=2.4. D=4 only. E= All
(Jawab:D REVIEW MANUAL FOR NEUROLOGY IN CLINICAL PRACTICE 4TH
EDITION no 73-6)

84. MRI images of a patient with complex partial seizures are shown in the figure. Top is T
1-weighted and bottom is T2-weighted imaging.

Which is the pathology shown in the scans?


A. Astrocytoma
B. Hamartoma
C. Herpes encephalitis
D. Mesial temporal sclerosis
(Jawab:D REVIEW MANUAL FOR NEUROLOGY IN CLINICAL PRACTICE 4TH
EDITION no 73-7)
85. Which of the following medications are helpful for patient with absence epilepsy?
1. Ethosuximide
2. Lamotrigine
3. Valproate
4. Carbamazepine
Select: A = 1,2,3. B = 1,3. C = 2, 4. D = 4 only. E = All
(Jawab:A REVIEW MANUAL FOR NEUROLOGY IN CLINICAL PRACTICE 4TH
EDITION no 73-8)
86. Which of the following are appropriate treatments for primary generalized tonic-clonic
seizures?
1. Valproic acid
2. Cacbamazepine
3. Phenytoin
4. Levetiracetam
Select: A= I, 2, 3. B = 1.3. C =2. 4. 0=4 only. E= All

(Jawab:E REVIEW MANUAL FOR NEUROLOGY IN CLINICAL PRACTICE 4TH


EDITION no 73-9)
87. Which of the following statements are true regarding vagal nerve stimulation for patients
with refractory epilepsy?
1. VNS results in complete control of seizures in about a third of patients
2. VNS reduces seizure frequency but not severity
3. VNS is predominantly used for generalized seizures
4. Patients with VNS need continued AED therapy
Select: A= 1,2,3. B = 1,3. C =2,4. D=4 only. E= All
(Jawab:D REVIEW MANUAL FOR NEUROLOGY IN CLINICAL PRACTICE 4TH
EDITION no 73-10)
88. Which of the following statements are true regarding Bell's palsy in pregnancy?
1. Bell's palsy is no more common in pregnancy
2. Women are most likely to develop Bell's palsy during the third trimester
3. Woman are most likely to develop Bell's palsy during the first trimester
4. Bell's palsy during pregnancy has no effect on the child
Select: A = 1,2,3. B = 1,3. C = 2, 4. D = 4 only. E = All
(Jawab:C REVIEW MANUAL FOR NEUROLOGY IN CLINICAL PRACTICE 4TH
EDITION no no 76-6)
89. A 77 year-old man presents with unilateral resting tremor. On examination, he also has
Stiffness of the right arm and leg, decreased facial expression, and defective postural
retlexes. There is no tremor or rigidity on the left Side. Which is the most likely
diagnosis?
A. Parkinson's disease
B. Vascular parkinsonism
C. Drug-induced parkinsonism
D. Progressive supranuclear palsy
(Jawab:A REVIEW MANUAL FOR NEUROLOGY IN CLINICAL PRACTICE 4TH
EDITION no 77-1)
90. A 75-year-old man presents with ataxia. frequent falls, and dementia. Examination shows
the cognitive disturbance plus rigidity and extended trunk and neck. He has dysarthria and
decreased eye movements to command. Which is the most likely diagnosis?
A. Multiple system atrophy
B. Parkinson's disease
C. Parkinson's-dementia complex
D. Progressive supranuclear palsy
(Jawab:D REVIEW MANUAL FOR NEUROLOGY IN CLINICAL PRACTICE 4TH
EDITION no 77-2)
91. A patient presents with parkinsonism and orthostatic hypotension and is found on
examination to have upgoing plantar responses. The diagnosis of MSA is made. What is
the expected response to treatment with L-dopa?
A. Treatment is contraindicated in MSA

B. Response is not expected so it should not be tried


C. Response may be less or short-lived but should be tried
D. Response is expected to be at least as good as with PD
(Jawab:C REVIEW MANUAL FOR NEUROLOGY IN CLINICAL PRACTICE 4TH
EDITION no 77-3)
92. Which of the following drugs have been implicated in causing drug-induced
parkinsonism?
1. Olanzapine
2. Valproate
3. Rispc,idone
4. Haloperidol
Select: A = 1,2,3. B = 1,3.C= 2.4. D=4 only. E = All
(Jawab:E REVIEW MANUAL FOR NEUROLOGY IN CLINICAL PRACTICE 4TH
EDITION no 77-4)
93. Your patient list for the day includes a 65-year-old man being referred to you because of
tremor. Knowing nothing else about the patient, which of the following is most likely?
A. Cerebellar tremor
B. Essential tremor
C. Huntington's disease
D. Parkinson's disease
(Jawab:B REVIEW MANUAL FOR NEUROLOGY IN CLINICAL PRACTICE 4TH
EDITION no 77-5)
94. A 54-year-old female presents with mild tardive dyskinesia. She is currently on an older
neuroleptic for a history of psychosis, although she has not had any psychotic episodes in
several years. Which is the next best course of management?
A. Tetrabenazine
B. Tocopherol
C. Change to an atypical neuroleptic
D. Discontinue all neuroleptic therapy
(Jawab:D REVIEW MANUAL FOR NEUROLOGY IN CLINICAL PRACTICE 4TH
EDITION no 77-7)
95. In evaluating a patient for a brachial plexopathy, which of the following clinical features
would suggest radiation-induced plexopathy rather than neoplastic infiltration?
1. Severe neuropathic pain
2. Upper plexus
3. Homer's syndrome
4. Lymphedema
Select: A = 1,2,3. B = 1,3. C = 2.4. D = 4 only. E = All
(Jawab:C REVIEW MANUAL FOR NEUROLOGY IN CLINICAL PRACTICE 4TH
EDITION no 81-4)

96. A 68-year-old man presents with severe shoulder pain which radiates into the neck., upper
chest, and into the medial aspect of the upper arm. He has weakness of the median and
ulnar-innervated intrinsic muscles of the hand. Which of the following are reasonable
differential diagnoses of this pain?
1. Carpal tunnel syndrome
2. Cervical radiculopathy
3. Monomelic amyotrophy
4. Brachial plexopathy
Select: A = 1,2,3. B = 1,3. C = 2, 4. D = 4 only. E = All
(Jawab:D REVIEW MANUAL FOR NEUROLOGY IN CLINICAL PRACTICE 4TH
EDITION no 81-9)
97. A 23-year-old female presents with 3 weeks of weakness most prominent distally,
dysesthesias and paresthesias. Examination is most remarkable for areflexia. Which is the
most likely diagnosis?
A. Acute inflammatory demyelinating polyradiculoneuropathy
B. Chronic inflammatory demyelinating polyradiculoneuropathy
C. Tick paralysis
D. Porphyria
(Jawab:A REVIEW MANUAL FOR NEUROLOGY IN CLINICAL PRACTICE 4TH
EDITION no 82-1)
98. A patient with carpal tunnel syndrome has marked increase in symptoms during
pregnancy. She already has tried wrist splints. Examination shows marked weakness of
the abductor pollicis brevis muscle. Which is the next best approach to treatment?
A. Anti-inflammatory treatment and continued use of wrist splints
B. Corticosteroid injection into the carpal tunnel
C. Corticosteroids by systemic administration
D. Endoscopic surgery
(Jawab:D REVIEW MANUAL FOR NEUROLOGY IN CLINICAL PRACTICE 4TH
EDITION no 87-2)
99. Which of the following statements is true regarding pregnancy and epilepsy?
1. AEDs increase the risk of birth defects
2. The greatest risk of birth defects is from exposure to AEDs io the first trimester
3. Folate supplementation prior to and during early gestation reduces the incidence of birth
defects
4. About 90% of epileptic women deliver nonnal babies
Select: A = 1.2.3. B = 1.3. C= 2.4. D= 4 only. E= All
(Jawab:E REVIEW MANUAL FOR NEUROLOGY IN CLINICAL PRACTICE 4TH
EDITION no 87-6)
100. A woman with complex partial seizures with secondary generalization is
contemplating pregnancy. The combination ofvalproate and oxcarbazepine completely

controls the seizures. You advise her about folate supplementation. How would you
advise her regarding medication management for the seizures?
A. Since she has good seizure control, the combination should be maintained
B. Monotherapy with OXC should be tried before becoming pregnant
C. Monotherapy with VPA should be tried before becoming pregnant
(Jawab:B REVIEW MANUAL FOR NEUROLOGY IN CLINICAL PRACTICE 4TH
EDITION no 87-7)
101. Wanita 30 tahun datang dengan nyeri di wajah yg dimulai sebulan lalu dengan rasa
aneh pada wajah.yang diikuti dengan sensasi tersengat listrik menjalar dari rahang hingga
mata kanan, seminggu kemudian merasakan hal yg sama disisi kiri .kadang bersamaan
dgn sisi yg kanan. Ia minum karbamazepin 600 mg/hari oleh dokter yg merujuknya untuk
kontrol nyeri. Pemeriksaan fisiknya dalam batas normal kecuali kelemahan pada lengan
dan kaki kiri. Pemeriksaan yg anda jalankan sebgai berikut kecuali:
a. MRI dengan dan tanpa kontras
b. EEG
c. Lab darah lengkap
d. Tes psikologis
e. Kadar karbamazepin serum
(Jawab : B, SANS V Nomor 11.)
102. Wanita 38 tahun penderita tekanan darah tinggi mengalami infark kapsula interna
kanan dan thalamus. Hal ini menyebabkan hemiparesis bermakana dan rasa tebal pada
sisi kiri tubuh. Segera setelah itu, timbul nyeri berat di lengan kiri, tungkai dan wajah.
Obat-obatan tidak mampu menangani nyeri tersebut. Prosedur operasi untuk menangani
sindroma nyeri thalamus adalah
a. implant alat intratechal drug delivery
b. topectomi frontal, cortekstomy parietal, atau cingulotomi
c. thalamotomi stereotaktik
d. deep brain stimulation
e. semua di atas benar
(Jawab : E, SANS V Nomor 38.)
Untuk pertanyaan 103-106 pilih jawaban dari kemungkinan di bawah
B. neuropraxia
C. axonotmesis
D. keduanya
E. bukan keduanya
103. membutuhkan regenerasi sel saraf untuk mengembalikan fungsinya
SANS V Nomor 62.)
104.

mengalami degenerasi Wallerian (Jawab : B, SANS V Nomor 63.)

105.

Perineurima intake. (Jawab : C, SANS V Nomor 64. )

(Jawab : B,

106. stabilisasi luka dengan transeksi anatomi komplit pada nerve (Jawab : D, SANS V
Nomor 65. )

107. Anak 6 tahun dengan penyakit Sturge-Weber mempunyai epilepsi intrakranial.


Kejangnya khas didominasi dengan gerakan tonik klonik pada eksteremitas kanan atas,
dengan kejang general yang tak tentu dan serangan status epilepsi.Lima obat dengan
berbagai kombinasi gagal mengontrol kejangnya.Kejang dimulai pada minggu pertama
dari kelahirannya, meningkat progresif dalm jumlah dan keparahan.Pemeriksaan klinis
mendapatkan hemiatrophi sisi kanan dengan hemianopia homonim kanan dan angioma
melewati pipi kiri dan temporal. EEG menunjukkan bankitan epilepsi multifokal,
predominan pada area central kiri.CT scan dan MRI sesuai dengan diagnosis penyakt
Sturge-Weber. Manajemen terbaik yang dilakukan adalah :
a. Kombinasi terbaru anti konvulsan
b. Reseksi lobus frontalis kiri
c. Hemispherectomy fungsional kiri.
d. Lobectomy temporal kiri
e. Callostomy corpus
(Jawab : C, SANS V Nomor 74.)
Untuk pertanyaan 108-113, petunjuk menjawab mengikuti dibawah ini :
A. Semua jawaban benar
B. Pilihan a dan c benar
C. Pilihan a,b,c benar
D. Pilihan d saja yang benar
E. Tidak ada jawaban benar
108. Manakah pernyataan yang benar berikut ini berkaitan Brain Stem Auditory Evoked
Potentials (BAEPs) (gambar 94A) ?
a. gelombang I dan gelombang II general pada nervus vestibulochlear
(VIII)
b. signal komputer rata=rata digunakan untuk mendeteksi signal
c. BAEPs berguna untuk menjaga ketulian sebagai komplikasi pembedahan
fossa posterior
d. BAEPs sangat resisten pada anestesia general
(Jawab : A, SANS V Nomor 94.)
109. Wanita, 47 tahun dengan neuralgia trigeminal yang telah diterapi carbamazepine
setelah 3 bulan terapi, nyeri berkurang, leukosit 2300 dan platelet 110.000> enzim hepar
sedikit meningkat. Tindakan dokter bedah saraf yang PALING SESUAI:
a. Menghentikan terapi carbamazepin dan menggantinya dengan dilantin
b. menghentikan carbamazepine dan menyarankan untuk operasi
c. Melanjutkan carbamazepine dengan tetap evaluasi hasil lab tiap 2-4
minggu
d. Melanjutkan carbamazepine tetapi dengan dosis yang diturunkan
e. Melanjutkan carbamazepine setelah mendiskusikan secara rinci pada
pasien, dengan monitor hasil lab tiap 3-6 bulan atau lebih awal apabila
ada keluhan.
(Jawab : E, SANS V Nomor 122.)
110. Wanita, 32 tahun menderita nyeri kepala hebat 2-4x/bulan, nyeri kepala bertahan
beberapa jam sampai beberapa hari. Nyerinya bilateral dan terasa menekan serta cekotcekot. Tidak ada mual atau

muntah yang menyertai nyeri kepala tetapi pasien terkadang mengalami fotofopia ringan.
Aktifitas sehari-hari seperti naik tangga, tidak memperparah nyeri kepalanya. Pasien
biasanya akan membatalkan kegiatannya jika sedang sakit kepala. Menurut International
Headache Society Classification 1988, diagnosa yang paling tepat untuk nyeri kepala ini
adalah:
a. cluster headache
b. Tension type headache episodik
c. Cluster migraine
d. Common migraine
e. Tension type headache
(Jawab : B, SANS V Nomor 125.)
111.

Manakah pernyataan di bawah ini yang SALAH tentang Moyarnoya disease?


a. Ditandai dengan stenosis arteri carotis interna
b. Biasanya pada anak akan disertai perdarahan
c. Berkaitan dengan defisiensi faktor VIII
d. Biasanya terdapat penebalan lapisan intima pada arteri di pankreas
e. Penyakit ini lebih banyak diderita wanita daripada pria
(Jawab : B, SANS V Nomor 132.)

112.

Manakah di bawah ini yang BUKAN manifestasi dari Friedreich ataxia?


a. familial
b. scoliosis
c. pes cavus
d. onsetnya pada usia muda sampai pertengahan 30 tahun
e. Perubahan status mental
(Jawab : D, SANS V Nomor 133.)

113.

Sindroma epilepsy lobus mesial temporalis berhubungan dengan hal berikut kecuali
a. Patologi yang paling sering ditemukan pada reseksi lobus temporalis
adalah hilangnya neuron hipokampus berat, disebut sclerosis
hipokampus
b. Sclerosis hipokampus terdiri dari hilangya neuron yang terberat di CA2
yang teringan di CA1(Sommers sector)
c. Reorganisasi sinaptik dari mossy fiber granule cell adalah gambaran
patologis yang berhubungan dengan sclerosis hipokampus
d. Kemungkinan berhasil paling baik dalam menghentikan kejang adalah
jika patologi fokal ditemukan di specimen temporal
(Jawab : B, Sans VI Nomor )
114. Berdasar pemeriksaan patologi klinis dari pasien dengan epilepsy lobus temporalis,
patogenesis sclerosis hipokampal yang paling mungkin
a. Selalu dimulai sebagai akibat kejang awal (c/ kejang demam, status
epileptikus)
b. Terjadi hanya pada kejang masa kecil
c. Membutuhkan cedera otak awal seperti status epileptikus dan atau
iskemi dan bukan merupakan konsekuensi dari epilepsy lobus temporalis
berulang
d. Merupakan konsekuensi dari epilepsy lobus temporalis berulang

(Jawab : C, Sans VI Nomor 48. )


115. Pada pasien dengan epilepsy intractable akibat lesi masa pada lobus temporalis
pernyataan berikut benar kecuali
a. Lesionektomi sama efektifnya mengkontrol kejang kronik seperti
lobektomi temporal anterior
b. Lokasi dari lesi masa pada lobus temporalis mempengaruhi jumlah
neuron hipokampus yang rusak
c. Terdapat kemungkinan besar bahwa lobektomi anterior akan
berhubungan dengan reduksi material spesifik pada tes dari memori
setelah operasi
d. Dengan riwayat kejang selama tahunan akan berhubungan dengan
semakin besarnya kerusakan neuron di hipokampus
(jawab : ?, Sans VI Nomor 50)
116. Sejumlah faktor neurotropik telah teridentifikasi memiliki peran yg potensial pada
perkembangan, maturasi, pengelolaan, atau regenerasi neuron pada CNS. Sebagian besar
data baik in vitro, atau lokalisasi anatomis dr faktor yg spesifik terhadap area CNS. Satu
faktor pertumbuhan, yg telah menunjukkan efek yg kuat, langsung, dan positif pada
neuron dopaminergik in vitro, dan berhubungan dengan penyakit Parkinson yaitu:
a. PDGF
b. Faktor neurotropik ciliary
c. Faktor neurotropik glial cell line-derived
d. IL-1
e. Faktor pertumbuhan saraf
(Jawab : C, Sans VI Nomor 102 )
117. Pernyataan di bawah ini benar sehubungan dengan spasme hemifasia (HFS),
KECUALI:
a. HFS muncul saat tidur
b. HFS biasanya disebabkan kompresi saraf fasialis pada jalur keluar arteri
serebelaris superior
c. Pada kasus HFS unik, angiogragm cerebral normal
d. Wlpn jarang, setiap pasien harus diMRI utk mengeksklusi tumor, AVM,
atau epidermoid
e. Karbamazepin dan feniotin tidak efektif untuk mengatasi gejala
(Jawab : B, Sans VI Nomor 105 )
118. Secara singkat, setelah memulai debulking intracapsular akustik neuroma
menggunakan pendekatan retromastoid pada posisi duduk, teknisi monitoring
memberitahu bahwa auditory evoked potential pada sisi tumor telah berubah. Perekaman
basal menunjukkan delay ringan dari gelombang I hingga III, tetapi gelombang I, III, dan
V ada dan dapat diulangi (reproducible). Kini gelombang I tidak diubah dalam hal latensi
atau amplitude, namun gelombang III dan V menjadi memanjang dalam hal latensi dan
penurunan amplitudo yang besar (75%). Apa yang PALING mungkin menyebabkan
kejadian tersebut?
a. Penekanan arteri auditori interna menyebabkan iskemia pada kohlea
b. Penekanan saraf VII pada fossa posterior
c. Problem teknis terhadap stimulator menyebabkan intensitas stimulasi
menurun dari 95 dari 30 dB

d. Peningkatan isofluran end tidal dari 0.4 hingga 0.8%


e. Semua diatas benar
(Jawab : D, Sans VI Nomor 126. )
119. Mengenai penggunaan somatosensory evoked potentials (SSEP) pada ruang operasi,
pernyataan di bawah ini yang BENAR?
a. Perekaman respon SSEP saraf medial dari permukaan-permukaan
korteks dapat digunakan untuk menentukan lokasi sulkus sentralis pada
area tangan
b. Respon SSEP yang reliabel dari struktur-struktur di sebelah rostral
medulla, tidak dapat direkam pada level bolus dan infus pentobarbital;
dimana menyediakan proteksi serebral terhadap iskemia/hipoksia
(contoh: cukup untuk memproduksi supresi-burst pada
elektroensefalogram (EEG)
c. Suplai aliran darah menuju jalur SSEP pada kapsula interna biasanya
berasal dari arteri lentikulostriata, sehingga SSEP dapat digunakan untuk
memonitor jalur maotoris pada area ini seara reliabel
d. Jalur spinal untuk SSEP saraf median adalah pada kolumna dorsalis,
mendapatkan suplai darah dari arteri spinalis anterior
e. Tidak ada yang benar
(Jawab : A, Sans VI Nomor 128. )
120. Sumber nyeri di bawah ini paling SEDIKIT berespon terhadap stimulasi kolumna
dorsalis, berasumsi bahwa semua metode mengkontrol nyeri telah gagal.
a. Nyeri stump pasca amputasi
b. Nyeri neuropatik yang berasal dari failed-back syndrome
c. Causalgia pada kaki setelah trauma akibat tembakan senjata
d. Nyeri iskemik yang terjadi akibat insufisiensi vaskular yang inoperabel
e. angina
(Jawab : A, Sans VI Nomor 140. )
Skenario untuk pertanyaan 121-122:
121. Laki-laki 65 tahun mengeluh kesulitan berjalan, terutama jarak jauh.Istrinya
mengatakan bahwa memori pasien melambat, tapi tidak lebih buruk dari dia
sendiri.Pasien harus buru-buru ke kamar mandi, namun tidak ada inkotinensia.Strength
normal, gait shuffling, reflek 2-3/4, tidak ada refleks patologis.
122. Work up apa yang harus dilakukan untuk pasien ini?
A. EMG dan konduksi saraf pada bilateral ekstrimitas inferior
B. MRI brain dan cervical spine
C. CT scan kepala dan isotope cisternogram
D. MRI lumbar spine
(Jawab : B, Sans VI Nomor 162. )
123. Faktor apa yang paling akurat dalam menentukan sukses shunting pasien dengan
gejala ventriculomegaly ....(tulisan terpotong) dan tanda dari hidrocephalus normal
pressure?
A. clinical history
B. Isotope cisternogram
C. MRI

D. Pungsi lumbal
(Jawab : A, Sans VI Nomor 163. )
124. Ketika perempuan diatas berumur 5 tahun, pola EEG berubah dan serangan menjadi
medically intractable, terutama terdiri dari axial tonic seizures, atonic seizures, dan
atypical seizures. Mana terapi pembedahan yang direkomendasikan untuk pasien ini?
A. temporal lobectomy
B. hemispherectomy
C. Corpus callosotomy
D. thalamotomy
E. multiple subpial transection
(Jawab : C, Sans VI Nomor 179. )
125. Memaki dan vokalisasi nyaring pada saat kejang dan menendang serta marah sering
berhubungan dengan kejang yang berasal dari
a. Neokorteks frontal
b. Neokorteks parietal
c. Region limbic mesial basal
d. Neokorteks oksipital
e. Neokorteks temporal
(Jawab : A, Sans VI Nomor 193. )
126.

stimulasi thalamic mungkin diindikasikan dengan pasien Parkinson dengan


a. disartria
b. tremor berat yang tidak berespon dengan pengobatan
c. disequlibrium
d. bradikinesia unilateral berat
e. semua di atas benar
(Jawab : B, Sans VI Nomor 200. )

127.

thalamotomi stereotaktik paling sukses dalam penanganan pasien dystonia yang mana
a. dystonia trunkal
b. dystonia cervical
c. dystonia facial
d. dystonia tungkai kontralateral
e. dystonia tungkai ipsilateral
(Jawab : D, Sans VI Nomor 201. )

128.

Berhubungan dengan Sindroma Tourettes (TS), manakah pernyataan yang benar


a. Tidak ada tempat terhadap terapi perilaku terhadap pengertian
neurobiologis dari TS
b. Haloperidol adalah obat pilihan
c. Stimulant dapat berguna untuk komponen obsesif-kompulsif dari
sindroma, tetapi hal ini dapat mengkambuhkan tics
d. Dyskinesia tardive bukan merupakan komplikasi dari neuroleptic pada
populasi pasien
e. Kebanyakan individu dengan TS tidak membutuhkan penanganan
(Jawab : E, Sans VI Nomor 202. )

129.

pernyataan dibawah ini tentang operasi epilepsy yang tidak benar:

a. penyebab tersering dari kejang parsial komplikata refrakter terhadap obat


adalah mesial temporal sclerosis
b. tidak mungkin membedakan munculnya kejang orbital frontal dan
medial temporal berdasarkan gejala klinis saja
c. banyak pasien dengan kejang parsial komplikata mengalami
pertumbuhan kelainan kejangnya selama remaja
d. temuan MRI berkolerasi tinggi dengan EEG pada kasus tipikal mesial
temporal sklerosis
(Jawab : C, Sans VI Nomor 241. )
130. A 23 year old woman with complex partial seizures was Initially treated with
phenytoin, then switched to carbamazepine, and is currently on levetiracetam after failing
the first two therapies. After a seizure-free period of six months upon starting
levetiracetam, she now has recurrence of 2-3 seizures/month. What is the most
appropriate next step in her management?
A. Evaluation for vagus nerve stimulation.
B. Add-on therapy with oxcarbazepine.
C. Switch to therapy with oxcarbazepine.
D Evaluation for resective epilepsy surgery
(Jawab : D , SANS VII, Nomor 1.)
131. A 32 yr old male with intractable seizures and a normal MRI scan undergoes a non
Invasive evaluation. Based on scalp EEG and seu:ure semiology, he had subdural
electrodes placed and seizures were localized to the left language-dommant
supplementary motor area Following surgical resectioo the patient will MOST likely
exhibit :
A. Temporary paresis on the left.
B. Left inferior quadrantopsia
C. Temporary Mutism
D. Anosmia and Finger agnosia
(Jawab : C , SANS VII, Nomor 2.)
132.

Patients with subcortical band heterotopia are characterized by:


A. Male predominance.
B. Subependymal giant cell astrocytomas.
C. Infantile spasms.
D. X-linked migrational disorder
(Jawab : D , SANS VII, Nomor 3.)

133.

The MOST common clinical feature of medial temporal lobe seizures is:
A. Visual aura.
B. Ictal Oral Automatism
C. Ictal bicycling movements
D. Ipsilateral dystonic posturing
(Jawab : B , SANS VII, Nomor 4.)

134.

Wests syndrome is BEST characterized by which of the following seizure type :


A. Primary and secondarily generalized
B. Generalized tonic clonic

C. Complex partial
D. Infantile Spasm
E. Atonic
(Jawab : D , SANS VII, Nomor 5.)
135.

Wests syndrome is best characterized by which of the following seizure type :


A. Primary and secondarily generalized
B. Generalized tonic clonic
C. Complex partial
D. Infantile Spasm
E. Atonic
(Jawab : D , SANS VII, Nomor 6.)

136. Which of the following structures is BEST described as lateral to the hippocampal
complex (hippocampus, subiculum and parahippocampal gyrus)?
A. Brain stem.
B. Ambient cistern.
C. Posterior cerebral artery.
D. Fusiform gyrus
E. Occulomotor nerve
(Jawab : D , SANS VII, Nomor 7.)
137. During the creation of a burr hole for a deep brain stimulator, an awake, sedated
patient develops the sudden onset of coughing, hypotension, and hypoxia. The clinical
presentation suggest which of the following complications:
A. Intracranial hemorrhage
B. Aspiration pneumonia
C. Air embolism
D. Tension pneumocephalus
(Jawab : C , SANS VII, Nomor 2.)
138. The most sensitive method for detecting carpal tunnel syndrome is
A. needle examination of the abductor pollicis brevis
B. needle examination of the first and second lumbricals
C. motor amplitude of the median nerve
D. motor distal latency of the median nerve
E. palmar sensory conduction time of the median nerve
Jawab : E
(NBR 2nd Edition 2005 chapter I : Neurosurgery, no. 30)
139. In the treatment of chronic pain, the undesirable effect(s) that is/are more common in
stimulation of the periaqueductal gray than the periventricular gray region is/are
I.
diplopia
II.
oscillopsia
III.
reduction of upgaze
IV.
sense of impending doom
A. I, II, III
B. I, III
C. II, IV

D. IV
E. all of the above
Jawab : E
(NBR 2nd Edition 2005 chapter I : Neurosurgery, no. 57)
140.

The transverse crest separates the


A. cochlear, facial, and superior vestibular nerves from the inferior vestibular nerve
B. cochlear and inferior vestibular nerve from the facial and superior vestibular nerve
C. facial and cochlear nerves from the superior and inferior vestibular nerves
D. facial, cochlear, and inferior vestibular nerves from the superior vestibular nerve
E. facial and inferior vestibular nerves from the cochlear and superior vestibular
nerves
Jawab : B
(NBR 2nd Edition 2005 chapter I : Neurosurgery, no. 99)

141.

Which of the following is true of hemifacial spasm?


A. Compression of the facial nerve by the superior cerebellar artery is the most
common operative finding.
B. Deafness is more common than permanent facial weakness as a complication of
microvascular decompression.
C. Men are more frequently affected than women.
D. Symptoms typically begin in the buccal muscles and move cranially.
E. The cure rate at 1 month after microvascular decompression is 95%.
Jawab : B
(NBR 2nd Edition 2005 chapter I : Neurosurgery, no. 100)
For questions 142 to 147, match the condition with the most appropriate treatment option.
Each treatment option may be used once, more than once, or not at all.
A. cingulotomy
B. dorsal root entry zone (DREZ) rhizotomy
C. morphine infusion
D. pallidotomy
E. sympathectomy
F. ventral rhizotomy

142. brachial plexus avulsion


Jawab : B
(NBR 2nd Edition 2005 chapter I : Neurosurgery, no. 190)
143. causalgia
Jawab : E
(NBR 2nd Edition 2005 chapter I : Neurosurgery, no. 191)
144. obsessive-compulsive disorder
Jawab : A
(NBR 2nd Edition 2005 chapter I : Neurosurgery, no. 192)
145. nociceptive cancer pain above C5
Jawab : C
(NBR 2nd Edition 2005 chapter I : Neurosurgery, no. 193)

146. Parkinson's disease


Jawab : D
(NBR 2nd Edition 2005 chapter I : Neurosurgery, no. 194)
147. spasmodic torticollis
Jawab : F
(NBR 2nd Edition 2005 chapter I : Neurosurgery, no. 195)
148.

Donor nerves that may be used for neurotization after brachial plexus avulsion include
I.
intercostal nerves
II.
spinal accessory nerve
III.
cervical plexus
IV.
phrenic nerve
A. I, II, III
B. I, III
C. II, IV
D. IV
E. all ofthe above
Jawab : E
(NBR 2nd Edition 2005 chapter I : Neurosurgery, no. 196)

149. Sakit bahu yang parah yang beberapa harikemudian diikuti oleh lumpuh pada tangan
di sekitarnya merupakan ciri khas dari
A. Sindrom Erb-Duchene
B. Sindroma PARSONAGE-TURNER
C. Sindroma DEJERINE-KLUMPKE
D. Sindroma Benedict
E. Bukan salah satu dari A sampai dengan D
(Jawab: B, INBR 7 Chapter 3 no.120)
150. Ny. X, 62 tahun menjalani dekompresi mikrovaskuler karena spasme hemifasial.
Pasca-bedah, pasien menderita ketulian ipsilateral total tanpa adanya defisit neurologis
lainnya Penyebab yg paling memungkinkan dari defisit ini adalah cederanya salah satu
pembuluh darah berasal dari
A. Arteri serebral belakang (PCA)
B. Arteri serebelar Atas (SCA)
C. Arteri serebellar Inferior Anterior (AICA)
D. Arteri serebellar bawah belakang (PICA)
E. Arteri vertebral
(Jawab: C, INBR 7 Chapter Neurosurgery no.19)
151. Salah satu prosedur paling dini yang dilakukan untuk penyakit Parkinson adalah ligasi
pembuluh darah yang mana?
A. Arteri koroidal anterior
B. Arteri koroidal belakang tengah
C. Arteri balik Heubner
D. Arteri tentorial Bernasconi dan Cassarini
E. Arteri lentikulostriata media
(Jawab: A, INBR 7 Chapter Neurosurgery no.23)

152. Stimulasi saraf vagal dicadangkan untuk memilih pasien dengan epilepsi. Mengapa
dilakukannya pada sisi kiri?
A. Untuk menghindari kemungkinan cedera pada saraf laringeal balik, yang mengikui
jalur ke arah kanan yang lebah rawan kerusakan
B. Untuk menghindari kemungkinan rusaknya saraf laryngeal atas dominan di sisi kanan.
C. Untuk menghindari kemungkinan rusaknya saraf kranial X, yang memasok jantung
terutama dari sisi kanan
D. Untuk menghindari kemungkinan cedera pada saluran torak
E. Lebih kecil peluang terjadinya paralisis dan seraknya urat suara dari kiri
(Jawab: C, INBR 7 Chapter Neurosurgery no.24)
153. Perawatan manakah yang menjadi pilihan utama untuk luka avulsi pleksus brakial
kronis dan yang membandel?
A. Kordotomi
B. Lesioning Zona Entri Akar Dorsal (DREZ)
C. Penempatan pompa morfin
D. Mielotomi garis tengah
E. Stimulasi otak dalam talamik lateral ventroposterior (VPL)
(Jawab: B, INBR 7 Chapter Neurosurgery no.25)
(SOAL INBR 7 Chapter Neurosurgery no.154 160)
A. Kordotomi
B. Stimulasi gray periakueduktal
C. Elektrokauteri trigeminal prakutanus
D. Simpatektomi
E. Talamotomi bilateral
F. Palidotomi
G. Mielotoni komisural
154. Disartria dan penurunan kognitif
no.48)

(Jawab: E, INBR 7 Chapter Neurosurgery

155. Hemiparesis, hemanopia homonimus (Jawab: F, INBR 7 Chapter Neurosurgery


no.49)
156.

Kutukan Ondine

(Jawab: A, INBR 7 Chapter Neurosurgery no.50)

157. Gangguan pergerakan mata, pupil dilatasi, rasa takut (Jawab: B, INBR 7 Chapter
Neurosurgery no.51)
158.

Sindroma HORNER

(Jawab: D, INBR 7 Chapter Neurosurgery no.52)

159.

Anestesia dolorosa

(Jawab: C, INBR 7 Chapter Neurosurgery no.53)

160. Lemah kaki, disestesia, disfungsi kandung Kemih (Jawab: G, INBR 7 Chapter
Neurosurgery no.54)
161.

Semua luka di bawah ini cocok untuk radiosurgeri stereostatis, KECUALI


A. Malformasi Arteriovenus 3 cm3 pada brainstem.

B. Karsinoma metastatis 1-cm frontal kanan dan 2-cm parietal kiri dari paru-paru.
C. Glioblastoma kambuhan pada lobe temporal kiri (2 cm3)
D. Kavermoma 1-cm dari inti kaudatus kanan yang sebelumnya mengalami
perdarahan.
E. Malformasi Arteriovenus talamik bilateral ( 3 cm3)
(Jawab: D, INBR 7 Chapter Neurosurgery no.72 )
162. Semua hal di bawah ini akan menurunkan penyebaran rasa sakit atau reaksi pasien
terhadap rasa sakit, KECUALI
A. stimulasi gray periakueduktal
B. Lobotomi pra-frontal
C. Singulotomi
D. Hipokampektomi
E. Kordotomi ventrolatera
(Jawab: D, INBR 7 Chapter Neurosurgery no.73)
163. Terapi-terapi bedah yang digunakan untuk Distonia biasanya meliputi semua hal di
bawah ini, KECUALI
A. Denervasi periferal
B. Pallidotomi
C. Talamotomi
D. Stimulasi kolom dorsal
E. Stimulasi korteks motorik
(Jawab: E, INBR 7 Chapter Neurosurgery no.89)
164. Semua prosedur bedah di bawah ini telah dilakukan untuk merawat penyakit
neuropsikiatris dan gangguan-gangguan keperilakuan, KECUALI
A. Faskikulotomi arkuate
B. Traktotomi subkaudate
C. Leukotomi limbik
D. Kapsulotomi depan
E. Singulotomi depan
(Jawab: A, INBR 7 Chapter Neurosurgery no.90)
165. Tn X., 45 tahun memiliki riwayat panjang epilepsi dari foci kejang yang berawal dari
korteks pra-motorik kanan dan menjalar ke dalam korteks motorik di sekitarnya. Kejangkejangnya tetap refraktoris kepada berbagai obat antiepilepsi, dan pasien telah dirujuk
kepada akhli bedah saraf untuk membahas mengenai pilihan-pilihan pembedahan. Catatan
hasil EEG mengungkapkan fokus kejang pada daerah pra-motorik yang menjalar ke
korteks motorik di sekitarnya. prosedur-prosedur pembedahan manakah di antara
prosedur-prosedur di bawah ini yang dapat dilaksanakan secara berbarengan selama
lesionektomi untuk menghindari cedera berat atas korteks motorik dan membantu
mengendalikan kejang-kejang pada pasien?
A. Topektomi
B. Lesionektomi terbatas
C. Stimulasi korteks motorik
D. Transeksi subpial multiple
E. Stimulasi saraf vagal
(Jawab: D, INBR 7 Chapter Neurosurgery no.98)

166. Manakah yang merupakan komplikasi yang secara neurologi paling berkaitan setelah
penempatan stimulator saraf vagal?
A. Mati rasa pada wajah
B. Bradikardia
C. Disponia
D. Hipotensi
E. Aritmia dengan durasi pendek
(Jawab: C, INBR 7 Chapter Neurosurgery no.99)
167.
Matching. Regarding surgical
treatment of Parkinson disease and its historical background, match the listed
procedures with the appropriate phrase(s) and benefits: Abandoned because (1 )
unpredictable results; (2) tremor did not improve; (3) bradykinesia did not improve; (4)
rigidity did not improve; (5) ipsilateral tremor persists; (6) side effects/resistance; (7)
only modest benefits Procedure:
a anterior choroidal artery ligation (1)
b anterodorsal pall idotomy (2,3)
c ventrolateral thalamotomy (3,4,5)
d L-dopa (6)
e transplantation Procedure currently pallidotomy of globus pallidus interna (GPI)
Beneficial for the following (percentage) (7)
f.dyskinesia is 90%
g bradycardia is 85 %
h rigidity is 75%
i tremor is 57 %
(Greenberg 15. Functional Neurosurgery no. 1)
168. True or False. The following symptoms improve after anterodorsal pallidotomy:
a tremor ipsilateral false
b rigidity true
c bradykinesia false
d ataxia false
e tremor contralateral false
(Greenberg 15. Functional Neurosurgery no. 2)
169. Ventrolateral thalamotomy can improve tremor; it cannot be performed bilaterally
because bilateral thalamotomy causes:
a). dysarthria
b). gait disturbance (Postoperative dysarthria and gait disturbance incidence is high.
(Greenberg 15. Functional Neurosurgery no. 3)
170. Complete the following about surgical treatment of Parkinson disease:
a The target today is the posteroventral pallidum
b Specifically the GPi-internal segment of the globus pallidus Which blocks the input
from the STN-Subthalamic nucleus
(Greenberg 15. Functional Neurosurgery no. 4)
171.

Complete the following about surgical treatment of Parkinson disease:

a Modern surgical treatment method is posteroventral pallidotomy


b Results in improvement in
dyskinesia of 90%
bradykinesia of 85%
rigidity of 75%
tremor of 57%
(Greenberg 15. Functional Neurosurgery no. 5)
172. Complete the following about surgical treatment of Parkinson disease:
a What was an early procedure for the treatment of Parkinson disease? ligation of the
anterior choroidal artery
b What are the mechanisms by which pallidotomy may work?
i). destroy GPi or
ii). Interrupt Pallidofugal pathways
iii). Reduce input into medial pallidum
c What is the target for the tremor treatment? ventralis intermedius nucleus (VIM) of the
thalamus
d True or False. Pallidotomy is primarily focused on the treatment of motor symptoms.
true
e What are the most common complications of pallidotomy? Hint: vhid
visual field deficit
hemiparesis
intracerebral hemorrhage
dysarthria
(Greenberg 15. Functional Neurosurgery no. 7)
173. True or False. Stimulation has attracted increasing interest in patients with Parkinson
disease who are refractory tomedical drug treatment. The deep brain stimulator (the
electrode) is placed in the following locations (there are three true answers):
a zona incerta false
b posterior ventral pallidum PV true
c substantia nigra SN false
d Forel field H false
e subthalamic nucleus STN true
f globus pallidus internus (GPi) true
(Greenberg 15. Functional Neurosurgery no. 8)
174. True or False. Indications for pallidotomyin Parkinsonism include
a refractory to drug therapy true
b drug-induced dyskinesia true
c rigidity true
d tremor false
e dementia false
(Greenberg 15. Functional Neurosurgery no. 9)
175. Ipsilateral hemianopsia is a contraindication to ventral pallidotomy because one of
the side effects of the procedure could be optic tract injury and would cause the patient
to be blind (Visual field defects could occur in 2.5% of patients; blindness could result).

(Greenberg 15. Functional Neurosurgery no. 10)


176. True or False. What are the benefits forthe patient from posteroventral pallidotomy
as done currently? Reduction in:
a dyskinesia true
b rigidity true
c bradykinesia true
d tremor true
(Greenberg 15. Functional Neurosurgery no. 11)
177. True or False. Common complications of pallidotomy (unilateral) include
a visual field deficit true
b dysarthria true
c hemisensory deficit false (Hemisensory deficit is not a common complication)
d hemiparesis (true)
(Greenberg 15. Functional Neurosurgery no. 12)
178. True or False. A spastic bladder will
a have high capacity and empty spontaneously . false
b have high capacity and empty with difficulty . false
c have low capacity and empty spontaneously. True (Low capacity and spontaneous
emptying are the hallmarks of the spastic bladder.)
d have low capacity and empty with difficulty. False
(Greenberg 15. Functional Neurosurgery no. 13)
179. True or False. The onset of a spastic bladder after spinal cord injury is
a immediate false
b delayed true (Delayed onset is typical because the acute phase of spinal shock is
hyporeflexic and hypotonic.)
c can occur at any time false
(Greenberg 15. Functional Neurosurgery no. 14)
180. True or False. The Ashworth score can grade severity of spasticity. The highest score
in this system is given when there is
a no increase in tone (full movement) false
b rigidity in all flexors false
c rigidity in all extensors false
d rigidity in flexion and extension true
(Greenberg 15. Functional Neurosurgery no. 15)
181. The Ashworth score is the clinical grading of the severity of spasticity
(Greenberg 15. Functional Neurosurgery no. 16)
182. What are the medications used in the treatment of spasticity?
baclofen
diazepam
dantrolene

progabide
(Greenberg 15. Functional Neurosurgery no. 17)
183. What are the nonablative procedures used for the treatment of spasticity?
intrathecal baclofen
intrathecal morphine
epidural electrical stimulation
(Greenberg 15. Functional Neurosurgery no. 18)
184. What are the ablative procedures with preservation of ambulation used for the
treatment of spasticity? Name one. motor point block, phenol nerve block, selective
neurectomy, percutaneous radiofrequency foramina! rhizotomy, Bischof myelotomy,
selective dorsal rhizotomy, stereotactic thalamotomy, or dentatotomy.
(Greenberg 15. Functional Neurosurgery no. 19)
185. What are the ablative procedures with sacrifice of ambulation used for the treatment
of spasticity? Name one. intrathecal injection of phenol, selective anterior rhizotomy,
neurectomy, intramuscular neurolysis, cordectomy, cordotomy
(Greenberg 15. Functional Neurosurgery no. 20)
186. True or False. Fibers that are more sensitive to radiofrequency rhizotomy are
a small unmyelinated sensory fibers true
b large myelinated alpha motor fibers false
(Greenberg 15. Functional Neurosurgery no. 21)
187. True or False. Spasticity can be treated with intrathecal baclofen pumps.
Complications are mainly
a pump underinfusion false
b wound complications false
c catheter complications true (Catheter complications may have a frequency of up to
30% in baclofen pumps.)
d drug resistance false
(Greenberg 15. Functional Neurosurgery no. 22)
188. What is another name for torticollis? wry neck
(Greenberg 15. Functional Neurosurgery no. 23)
189. What muscle is usually affected in spasmodic torticollis? Sternocleidomastoid
(Greenberg 15. Functional Neurosurgery no. 24)
190. What are the surgical procedures used for the treatment of spasmodic torticollis?
a stimulate dorsal cord
b inject botulinum toxin
c cut rhizotomy
d coagulate Forel's Hl
(Greenberg 15. Functional Neurosurgery no. 25)

191. What artery is most commonly implicated in the torticollis of the eleventh nerve
origin? Vertebra
(Greenberg 15. Functional Neurosurgery no. 26)
192. True or False. Hemifacial spasm (HFS) and spreads to the upper half of the face.
false (starts with the starts from the lower half of the face orbicularis oculi)
(Greenberg 15. Functional Neurosurgery no. 27)
193. What distinguishes HFS from facialmyokymia (FM)?
a HFS is unilateral
b Blepharospasm is bilateral
(Greenberg 15. Functional Neurosurgery no. 28)
194. What is the only other involuntary palatal myoclonus movement disorder besides
HFS that persists during sleep? palatal myoclonus
(Greenberg 15. Functional Neurosurgery no. 30)
195. Complete the following statements about neurovascular compression syndromes:
a On what side is HFS more common? left
b What is the age and gender predilection? Women, after the teen ages
c What is the most commonly involved artery? AICA
d True or False. Carbamazepine and phenytoin are generally effective treatment. false
e What is the material used as a cushion in the microvascular decompression (MVD)7
? lvalon, polyinyl formyl alcohol foam
(Greenberg 15. Functional Neurosurgery no. 31)
196. True or False. The vessel most commonly associated with hemifacial spasm is
a posterior inferior cerebel lar artery (PICA) false
b superior cerebellar artery (SCA) false
c anterior inferior cerebellar artery (AICA) true
d posterior cerebral artery (PCA) false
e vertebral artery false
f basilar artery false
(Greenberg 15. Functional Neurosurgery no. 32)
197. Hemifacial spasm
a is caused by compression at the root entry zone
b of the facial nerve
c by the AICA
d This does not cause ephaptic conduction but
e produces kindling
f and synkinesis
(Greenberg 15. Functional Neurosurgery no. 33)
198. Synkinesis is a phenomenon where
a stimulation of facial nerve of the one branch
b results in delayed discharges

c through another branch


(Greenberg 15. Functional Neurosurgery no. 34)
199. True or False. Postoperatively after microvascular decompression for hemifacial
spasm the patient can expect
a immediate cessation of facial spasms false
b reduction starting 2 to 3 days later true
c better results the longer the patient has had HFS false
d better results the older the patient is false
e complete resolution of spasms eventually true (in 81 -93% of patients)
f that relapse may still occur even if free of spasms for a full 2 years false (relapse after
2 years only 1 %)
(Greenberg 15. Functional Neurosurgery no. 35)
200. Complications of hemifacial spasm (HFS} surgery include the following: Hint:
hemifacial s
hoarseness
elderly do less well
meningitis (aseptic)
ispsilateral hearing loss
facial weakness
ataxia
CSF rhinorrhea
incomplete relief
aseptic meningitis
lip (perioral) herpes
swallowing (dysphagia)
(Greenberg 15. Functional Neurosurgery no. 36)
201. Complete the following statements about hyperhidrosis:
a Due to overactivity of the eccrine sweat.lands.
b These glands are under control of the sympathetic nervous system
c The neurotransmitter is acetylcholine
d Most sympathetic end organs are adrenergic
e Some cases warrant surgical sympathectomy
(Greenberg 15. Functional Neurosurgery no. 37)
202. Name five indications for upper extremity (UE) sympathectomy. Hint: "crash" the
sympathetic ganglia
causalgia major primary
Raynaud disease
intractable angina
shoulder-hand syndrome
hyperhidrosis
(Greenberg 15. Functional Neurosurgery no. 38)
203. Complete the following statements about sympathectomy:
a What is the level for cardiac sympathectomy? from stellate ganglion

b What is the level for UE sympathectomy? through T7 ganglion second thoracic


ganglia
c What is the level for lumbar sympathectomy? L 2 and L3 sympathetic ganglia
d What is the most commonly used approach for lumbar sympathectomy?
Retroperitoneal
(Greenberg 15. Functional Neurosurgery no. 39)
204. What are the complications of UE sympathectomy?
pneumothorax
intercostal neuralgia
spinal cord injury
Horner syndrome
(Greenberg 15. Functional Neurosurgery no. 40)
205. Complete the following statements about pain:
a Three types of pain are:
nociceptive
deafferentation
sympathetically maintained
b Two types of nociceptive pain are:
somatic
visceral
c Two sites of electrical stimulation for pain in deep brain are:
periaqueductal gray
periventricular gray
(Greenberg 16. Pain no. 1)
206. Complete the following statements about craniofacial pain syndromes:
a Tic convulsif is geniculate neruralgia plus hemifacial spasm
b Ramsay-Hunt syndrome is postherpetic geniculate neuralgia
c Tolosa Hunt syndrome is superior orbital fissure inflammation
d Raeder's neuralgia is paratrigeminal neuralgia
(Greenberg 16. Pain no. 2)
207. Complete the following regarding primary otalgia:
a It may have its origin from which nerves? 5 7 9 1 0 and occipital nerves
b Cocainization of the pharynx producing pain relief suggests glossopharyngealneuralgia
instead of primary otalgia
c Treatment includes:
i). medicine tegretol, dilanting, and baclofen
ii). surgical procedures of decompression by microvascular decompression (MVD) ,
nerve fibers, nervus intermedius, ninth and tenth CN.
(Greenberg 16. Pain no. 3)
208. Characterize trigeminal neuralgia (TGN):
a The incidence is 4/1 00,000
b The percentage of multiple sclerosis (MS) patients who have TGN is 2%
c The percentage of bilateral TGN patients is 18 %

d It is pathophysiologically caused by ephaptic transmission from large myelinated A


fibers to poorly myelinated A delta and C fibers
e Caused
i). most commonly by superior cerebellar artery
ii). Or persistent primitive trigeminal artery
iii). Or basilar artery
(Greenberg 16. Pain no. 4)
209. Complete the following statements about craniofacial pain syndromes:
a What should the neurologic exam be in a patient with trigeminal neuralgia?
entirely normal
b How effective is Tegretol? pain relief in 69%
c What if Tegretol has no effect? The diagnosis of trigeminal neuralgia is suspect.
d What is the second drug of choice for trigeminal neuralgia? baclofen (Lioresal)
e The two special cautions needed with theuse of this medication are as follows:
i.
It may be teratogenic
ii.
Don't stop abruptly
(Greenberg 16. Pain no. 5)
210. Medicines for trigeminal neuralgia include the following:
amitriptyline (old)
baclofen
carbamazepine, clonazepam,
capsaicin
Dilantin
Elavil
gabapentin
Lamictal
(Greenberg 16. Pain no. 6)
211. Basis upon which percutaneous procedures treat trigeminal neuralgia:
a They destroy nociceptive fibers
b which are A and C
c and preserve touch fibers
d which are A and
(Greenberg 16. Pain no. 7)
212. Which treatment procedure is mosthelpful in trigeminal neuralgia in multiple
sclerosis patients? percutaneous techniques (MVD (microvascular decompression] does
not work well for MS [multiple sclerosis) patients with TGN[trigeminal neuralgia)).
(Greenberg 16. Pain no. 8)
213. State the benefits of percutaneous microcompression (PMC):
a Patient can choose to avoid major surgery
b With multiple sclerosis and trigeminal neuralgia treatment
i.
Which procedure is best? balloon PMC
ii.
Does it respond to microvascular decompression? not well
c Occurances of intraoperative hypertension are less with PMC than with radiofrequency

d Reports of intracranial hemorrhage none reported with PMC


(Greenberg 16. Pain no. 9)
214. Considering trigeminal neuralgia and microvascular decompression (MVD):
a True or False. It is appropriate for an older age group. false (not to be used on
persons over 65)
b True or False. It may produce anesthesia dolorosa. false (It does not occur with MVD.)
c It has a mortality rate of 1 %
d It has a major neurologic morbidity of 1 to 10%
e It has a failure rate of 20 to 25 %
f True or False. It is the procedure of choice in MS patients. false (MS patients do not
respond to MVD.)
g What is the procedure of choice in MS patients? percutaneous microcompression
(PMC} (i. e., balloon)
h What is the recurrence rate in MS patients? 50% in 3 years with percutaneous
techniques
(Greenberg 16. Pain no. 10)
215. Considering TGN and the benefits of stereotactic radiosurgery:
a Complete pain relief is achieved in 65%
b There is significant pain reduction in an 15 to 30 additional %.
(Greenberg 16. Pain no. 11)
216. Regarding TGN and electrode positioning:
a Positioning for percutaneous approach
i.
lip: lateral to lip 2 to 3 cm
ii.
eye medial aspect of pupil
iii.
ear: 3 cm anterior to external auditory meatus
b X-ray landmarks
i.
anteroposterior (AP}-submental vertex, aim for foramen ovale
ii.
lateral x-ray aim for 10 mm below floor of sella along clivus
(Greenberg 16. Pain no. 12)
217.
a
b
c
d
e

Characterize complications of radiofrequency trigeminal rhizotomy:


masseter weakness 24%
anesthesia dolorosa 4%
neuroparalytic keratitis 4%
oculomotor paresis 2%
How would you identify pterygoid muscle weakness?
i.
ask patient to open mouth
ii.
chin deviates to side of weak pterygoid
(Greenberg 16. Pain no. 13)

218.
a
b
c
d

Describe microvascular decompression (MVD) complications


mortality 0,22 to 2 %
morbidity 1 to 10 %
hearing loss 3%
infarction 0,6%

e success rate 75-80%, approximately


(Greenberg 16. Pain no. 14)
219. Concerning glossopharyngeal neuralgia:
a Pain is located in: base of tongue = glosso throat = pharyngea
b Other symptoms
hypotension-vagus
syncope
cardiac arrest
(Greenberg 16. Pain no. 15)
220. Describe glossopharyngeal neuralgia:
a The incidence is 1 in 1, 775,000 persons.
b. Pain occurs in throat, base of tongue, ear, neck
c Treatment:
i.
medicine : cocainization
ii.
surgery: microvascular decompression
iii.
section of ninth and upper third of tenth nerve
(Greenberg 16. Pain no. 16)
221.
a
b
c
d
e

Concerning geniculate neuralgia:


Pain is located deep in the ear, eye, cheek
It is called prosopalgia
If there are herpetic lesions this is called Ramsey-Hunt syndrome
If combined with hemifacial spasm it is called tic convulsif
Treatment
i.
medicine: same as trigeminal neuralgia
ii.
surgery: MVD of seventh nerve
iii.
What vessel is involved? AICA-compressing sensory and motor roots of
seventh nerve
(Greenberg 16. Pain no. 17)

222. Complete the following statements about herpes zoster:


about herpes zoster:
a The etiologic agent is herpes varicella zoster virus
b Involves the eye in 10%
c Pain lasts 2 to 4 weeks
d Long-term pain persists in 10%
e called post herpetic neuralgia
f Vesicles and pain runs in the distribution of the dermatome not the peripheral nerve
g Treatment is with capsaicin amitriptyline
(Greenberg 16. Pain no. 18)
223. Complete the following statements about postherpetic neuralgia:
a With an acute attack of herpes zoster, you may treat with epidural or intercostal
injection
b For acute treatment use
i.
Acyclovir
ii.
Valacyclovir

c For postherpetic neuralgia use


i.
Z ostrix (capsaicin)
ii.
Gabapetin (neurotin)
iii.
Elavil (Amitriptilin )
d Start treatment with lidocaine patches
(Greenberg 16. Pain no. 19)
224. Usual maximum oral narcotic dose tolerated is MS contin (up to 300 to 400
mg/day)
(Greenberg 16. Pain no. 20)
225. Name three intracranial ablative procedures to treat the following pains:
a cancer pain: medial thalamotomy (stereotactic procedure used for nociceptive cancer
pain)
b head, neck, face pain: stereotactic mesencephaloton lesion 5 mm lateral to aqueduct at
level of inferior colliculus; diplopia may occur
c suffering from pain: cingulotomy-bilaterally (modifies affect use MRIrecurs in
approximately 3 months)
(Greenberg 16. Pain no. 21)
226. Matching. Match the procedure and its application (some have more than one).
Applications for pain from: ( spinal cord injuries; (1) post laminectomy pain; (2) pelvic
pain with incontinence; (3) at or below C 5; (4) head, face, neck, upper extremity; (5)
bilateral below diaphragm; (6) causalgia; (7)bilateral below thoracic dermatomes; (8)
avulsion injuries; (9) not for cancer pain
Procedure:
a stereotactic mesencephalotomy (5)
b cordotomy (4)
c spinal intrathecal (6)
d sacral cordotomy (3)
e sympathectomy (7)
f commissural myelotomy (8)
g dorsal root entry zone (DREZ) (1,9,10)
h spinal cord stimulator (2,10)
(Greenberg 16. Pain no. 22)
227. Concerning cordotomy
a Your objective is to interrupt the fibers ofthe lateral spinal thalamic tract on the
contralateral on the side to the pain.
b Cordotomy is the procedure of choice for unilateral pain below the C5dermatome.
c Two ways to perform cordotomy are:
i.
Open
ii.
percutaneus
d Loss of automatic breathing can occur after bilateral cordotomy and is called
Ondine's curse
e What is the cutoff percentage on pulmonary function test before patients can undergo
cordotomy? 50%

(Greenberg 16. Pain no. 23)


228. Complete the following statements about pain procedures:
a What kind of patients are candidates for cordotomy? terminally ill patients
b On which side should the cordotomy be performed? contralateral to the pain
c What happens to impedance as the needle penetrates the cord? it jumps from 300 -500
ohms to 1200 - 1500 ohms
d What response should stop cordotomy from being performed? muscle tetany
uponstimulation
e If you look at the eye what will you learn? if there is a Horner's syndrome
ipsilaterally the procedure is satisfactory
f What percent will have pain relief? 94%
(Greenberg 16. Pain no. 24)
229. Concerning commissural myelotomy:
a What is the indication for commissural myelotomy? bilateral or midline pain
b What is the rate of complete pain reliefafter commissural myelotomy? 60%
c What is the special requirement for intrathecal morphine? preservative-free 0.9%
saline
(Greenberg 16. Pain no. 25)
230. Regarding central nervous system (CNS)narcotic administration:
a Requirement for implantation of a morphine pump is preoperative testing dose
b bolus infusion can shorten the delay time for a morphine pump to function; otherise
therelief may not occur for days
c Is meningitis common after pump placement? No
d Is respi ratory failure common after pump placement? No
(Greenberg 16. Pain no. 26)
231. Concerning spinal cord stimulation:
a Site of spinal cord stimulation is the dorsal column
b
i.
The most common indication is postlaminectomy pain syndrome
ii.
It is not usually indicated for cancer pain
c Two kinds of electrodes:
i.
plate ike
ii.
wire like
(Greenberg 16. Pain no. 27)
232. Regarding deep brain stimulation:
a Periventricular stimulation will be beneficial for nociceptive pain.
b A lesion at the dorsal root entry zone can help phantom limb pain.
c Rate of recurrence after thalamotomy forpain is 60% in 6 months
(Greenberg 16. Pain no. 28)
233. Complete the following statements about causalgia:
a Triad to diagnose causalgia:
i.
autonomic dysfunction

ii.
burning pain
iii.
trophic changes
b What is the cause of major causalgia? high-velocity missile injury
c Allodynia is pain induced by non-noxious stimulus
d Signs of causalgia:
i.
tapered fingers
ii.
hands are and cold and moist
iii.
touching causes pain
iv.
also known as allodyni
e Current name for causalgia is complex regional pain syndrome (CRPS)
(Greenberg 16. Pain no. 29)
234. Complete the following statements about causalgia:
a Medical treatment for causalgia uses tricyclic antidepressants
b A common agent used for intravenous injection for causalgia is guanethedine
c Surgical sympathectomy may relieve thepain of causalgia in 50 %
(Greenberg 16. Pain no. 30)
235. Is stereotactic radiosurgery {SRS) useful for:
a venous angiomas? no
b cavernous angiomas? No
(Greenberg 18. Radiation Therapy no. 1)
236. Complete the following about stereotactic surgery
a For most cases what is the optimal treatment for vestibular schwannoma? surgery
b What alternative is available? SRS
c When would the alternative for the patient be considered?
i.
poor medical condition
ii.
older age group
(Greenberg 18. Radiation Therapy no. 2)
237. With stereotactic radiosurgery:
a Accuracy is never better than __ 0.6 mm
b If embolization is used what precaution is advised before SRS? wait 30 days between
procedures
c What dose is optimal for an arteriovenous malformation (AVM)? 1 0 to 15 Gy to
periphery of AVM
d What dose is optimal for tumors? 1 0 to 15 Gy with tumor in the 80% isodose line
e What dose is optimal for metastatic tumors? 15 Gy to center of tumor in the 80%
isodose line
(Greenberg 18. Radiation Therapy no. 3)
238. Complete the following regarding theresults, in percent, of SRS obliterationof:
a AVM
i.
AVM at 1 year 46 to 61 %
ii.
AVM at 2 years 86 %
iii.
under 2 cm 94 %
iv.
over 2.5 cm 50 %

b acoustic tumor
i.
decreased in size 44 %
ii.
stabilized in size 42 %
iii.
increased in size 14 %
c local metastatic control 88%
(Greenberg 18. Radiation Therapy no. 4)
239. What is advised if, after SRS, an AVM persists after 2 to 3 years? may re-treat with
SRS again
(Greenberg 18. Radiation Therapy no. 5)
240. Is there any difference in outcome toSRS by radio-resistant versus radio-sensitive
tumors? no
(Greenberg 18. Radiation Therapy no. 6)
241. Which has a better response, supra- o infratentorial metastases? supra
(Greenberg 18. Radiation Therapy no. 7)
242. Which premedication is given before SRS? steroids and phenobarbital
(Greenberg 18. Radiation Therapy no. 8)
243. During the latency period is there a higher incidence of hemorrhage from AVM? no,
approximately 3 to 4%/year
(Greenberg 18. Radiation Therapy no. 9)
244. For interstitial brachytherapy:
a How much radiation is given? 60 Gy
b To what area? a volume that extends 1 cm beyond the contrast enhancing tumor
c At what rate? 40 to 50 c Gy/h
d For how many days? 6
e What is the radiation amount that will cause tumor growth to stop? 30 c Gy/h
f With this protocol what percent of patients develop symptomatic radiation necrosis?
40%
(Greenberg 18. Radiation Therapy no. 10)
245. True or False. Indications for stereotacti surgery include:
a biopsy of multiple lesions . true
b brachytherapy implants . true (catheter placement for brachytherapy)
c treatment of chronic pain . true (electrode placement for pain, seizures)
d gamma knife radiosurgery. true (lesion generation for trigeminal pain)
e biopsy of a deep cerebral lesion . true
f hematoma evacuation . true (evacuation of intracerebral hemorrhage,cystic fluid)
g localization of lesion for open craniotomy. true
(Greenberg 19. Stereotactic Surgery no. 1)
246. True or False. Stereotactic biopsy contraindications include:
a coagulopathy . true

b multiple lesions . false (Multiple lesions are anindication for stereotactic biopsy.)
c brain stem lesions . false (A brain stem lesion is an indication for stereotactic
biopsy.)
d inabi lity to tolerate general anesthesia . false (can usually tolerate local anesthesia)
e thrombocytopenia . true (Platelets below 50,000 are an absolute contraindication to
biopsy.)
f inability to cooperate for biopsy. false (may do stereotactic biopsy under general
anesthesia)
(Greenberg 19. Stereotactic Surgery no. 2)
247. True or False. The yield rate for stereotactic biopsy is:
a higher for enhancing lesions than for nonenhancing . true
b lower for enhancing lesions than nonenhancing . false
c enhancing and nonenhancing lesions have equal yield rates. False
(Greenberg 19. Stereotactic Surgery no. 3)
248. True or False. The most common complication of stereotactic surgery is:
a hemorrhage. true (Most are too small to be clinically significant. The hemorrhage rate
is higher in AIDS and in central nervous system lymphoma.)
b infection . false
c inabi lity to localize lesion .false
d inabil ity to provide sufficient tissue qual ity/ quantity for biopsy .false
(Greenberg 19. Stereotactic Surgery no. 4)
249. Regarding stereotactic biopsy:
a True or False. The risk for major complication due to stereotactic biopsy is higher
in patients with multifocal high grade gliomas than in patients with AIDS. false
b Relative risk for patients that are
i.
immune compromised 0-12 %
ii.
nonimmune compromised 0-3 %
iii.
or have glioma 3 %
(Greenberg 19. Stereotactic Surgery no. 5)
250. True or False. The peripheral nervou system includes:
a spinal nerves . true
b all cranial nerves .false
c cranial nerves II I-XII . true
d cervical, brachial, lumbosacral plexus .true
(Greenberg 20. Peripheral Nerves no. 1)
251. True or False. Upper motor neuron paralysis includes:
a clorius . true
b hyperactive reflexes . true
c muscle spasms .true
d atrophy . false
e fasciculations . false (Choices d and e are characteristic of lower motor neuron
paralysis.)
(Greenberg 20. Peripheral Nerves no. 2)

252. List the 11 muscles of the shoulder and arm and nerves to test:
a muscle trapezius, nerve CN Xl spinal accessory , roots, C3,4
b muscle serratus anterior, nerve long thoracic, roots C 5,6,7
c muscle suprasupinatus , nerve suprascapular, roots C 4,5,6
d muscle infraspinatus , nervesuprascapular, roots C 5,6
e muscle rhomboids, nerve dorsal scapular, roots C 4,5
f muscle pectoralis minor, nerve anterior thoracic (med) AKA pectoral nerve, roots C 7,8
g muscle pectoralis major , nerve (lat. Anterior thoracic anterior thoracic med) AKA
pectoral nerve, roots C4,5,6,7,8,
h muscle latissimus dorsi, nerve thoracodorsal, roots C 5,6,7,8
i muscle deltoid, nerve axillary, roots C 5,6,
j muscle brachialis , nerve musculocutaneus, roots C 5,6
k muscle biceps, nerve musculocutaneus, roots C 5,6
(Greenberg 20. Peripheral Nerves no. 3)
253. List 11 muscles of the shoulder and arm, their nerve, and their action.
a muscle trapezius , nerve CNXl , action shrug shoulder
b muscle serratus anterior , nerve long thoracic , action forward shoulder thrust
c muscle supraspinatus , nerve supracapsular, action abduct arm 90 degrees
d muscle infraspinatus , nerve supracapsular , action backhand tennis shot
e muscle rhomboids , nerve dorsal scapular , action abduct scapulae
f muscle pectoralis minor , nerve pectoral nerve medial , action adduction arm
g muscle pectoralis major , nerve pectoral nerve lateral and medial , action adduction
arm and push arm forward
h muscle latissimus dorsi , nerve thoracodorsal , action adduct arm, ladder climb, cough
j muscle deltoid , nerve axillary, action abduct arm > 90 degrees
k muscle brachialis , nerve musculocutaneous , action flex forearm
l muscle biceps , nerve musculocutaneous , action flex and supinate forearm
(Greenberg 20. Peripheral Nerves no. 4)
254. True or False. The suprascapular nerve innervates:
a teres major . false-subscapular nerve (C5 -7)
b teres minor false-axillary nerve (C4 - 5)
c infraspinatus true
d supraspinatus true
(Greenberg 20. Peripheral Nerves no. 5)
255. The suprascapular nerve contains roots from C 4,5,6
(Greenberg 20. Peripheral Nerves no. 6)
256. Describe the latissimus dorsi muscle:
a function
ladder climbing
cough
adductor-together with pectoralis
b nerve thoracodorsal nerve
c cord posterior cord

d roots C 6,7,8
(Greenberg 20. Peripheral Nerves no. 7)
257. True or False. The deltoid muscle:
a abducts arm 0 to 90 degrees false (The arm is abducted 0 to 90 degrees by the
supraspinatous muscle.)
b abduct arm > 90 degrees .true
c is innervated by the axillary nerve. true
d rotates the arm out . false (Arm is rotated out by the infraspinatus muscle.)
(Greenberg 20. Peripheral Nerves no. 8)
258. True or False. The abductor pollicis longus
a is innervated by the median nerve false
b is innervated by the radial nerve true
c is innervated by the ulnar nerve false
d is innervated by the posterior interosseous nerve true (The posterior interosseus nerve
is a continuation of the radial nerve in the forearm.)
(Greenberg 20. Peripheral Nerves no. 9)
259. True or False. The median nerve is responsible for the following movements of the
thumb:
a adduction . false (served by ulnar nerve)
b abduction . true
c extension .false (served by radial nerve)
d flexion .true
e opposition . true
(Greenberg 20. Peripheral Nerves no. 10)
260. Complete the following about the movements of the thumb:
a Actions of nerves to the thumb:
i median nerve, Hint: FAO
Flexion action muscle flexor pollicis brevis and longus root C 8, T1 , median
A-action abduction, muscle abductor pollicis brevis , root C 8, T1 , median
O-action opposition, muscle opponens pollicis, root C8, T1
ii ulnar nerve
action adduction, muscle adductor pollicis , root C 8, T1
iii. radial nerve
action extension , muscle extensor pollicis brevis and longus , root C7, C8
b Plane of movement for the thumb:
i.
extension is plane of palm
ii.
flexion is plane of palm
iii.
adduction is perpendicular to palm
iv.
abduction is perpendicular from palm
v.
opposition is across the palm
(Greenberg 20. Peripheral Nerves no. 11)

261. Complete the fol lowing about peripheral nerves: Hint: fosis pdstp follow our sign.
it says "please don't spoil the plants"
femoral
obturator
superior gluteal
inferior gluteal
sciatic (trunk)
peroneal (trunk)
deep peroneal
superficial peroneal
tibial
pudendal
(Greenberg 20. Peripheral Nerves no. 12)
262. Name the nerves of the lower extremities and the roots that form them:
femoral, 1,2,3
obturator, 2,3
superior gluteal, 4, 5, 51
inferior gluteal, 5, 51, 52
sciatic, 5, 51, 52
peroneal, 4, 5, 51
deep peroneal, 4, 5
superficial peroneal, 5, 51
tibial, 4, 5, 51, 52, 53
pudendal, 52, 53, 54
(Greenberg 20. Peripheral Nerves no. 13)
263. Name the nerves of the lower extremities and the muscles and function of the
muscles they serve:
a nerve femoral, muscle iliopsoas, quadriceps femoris, sartorius , function flex hip
a nerve , muscle , function
a nerve obturator , muscle adductor , function adduct thigh
a nerve superior gluteal , muscle gluteus medius , function abduct thigh
a nerve inferior gluteal , muscle gluteus maximus , function flex leg
a nerve sciatic trunk , muscle biceps femoris, semi tendenosis, semi membranosis,
function extend thigh
a nerve deep peroneal, muscle tibialis anterior, extension hallucis longus (EHL),
function great toe extension, foot dors, flexion,
a nerve superficial peroneal , muscle peroneus longus , function plantar flexion foot and
toes
a nerve tibial , muscle posterior tibial, gastrocnemius, soleus, flexor hallacis longus
(FHL) , function plantar flex foot and toes,
a nerve pudendal , muscle perineal, sphincters, function voluntary contraction of pelvic
floor
(Greenberg 20. Peripheral Nerves no. 14)

264. True or False. The gluteus maximus muscle:


a abducts thigh true (The gluteus maximus abducts thigh in a prone position.)
b adducts thigh false (thigh-adduction -the obturator externus muscle and pectineus
muscle)
c medially rotates thigh false (thigh-medial rotation -the gluteus medius and minimus
muscle)
d externally rotates thigh false (thigh-external rotation - the obturator externus muscle)
e is innervated by superior gluteal nerve false (the gluteus maximus is innervated by
the inferior gluteal nerve)
(Greenberg 20. Peripheral Nerves no. 15)
265. True or False. The tibialis anterior muscle is responsible for foot:
a dorsiflexion true
b plantar flexion false (plantar flexion-soleus muscle, gastrocnemius muscle)
c eversion false (eversion-peroneus longus and brevis muscles)
d inversion false (inversion-posterior tibialis muscle)
(Greenberg 20. Peripheral Nerves no. 16)
266. Complete the following about the function of peripheral nerves:
a The function of extension of the great toe is served
by: muscle extensor hallucis longus , root L 5
b The function of foot dorsiflexion is served
by: muscle tibialis anterior , root L4
c Which is the best L 5 muscle? Hint: The letter E is the 5th letter in the alphabet.
extensor hallucis longus.
(Greenberg 20. Peripheral Nerves no. 17)
267. True or False. The extensor hallucis longus muscle
a is the best L 5 muscle . true
b extends great toe true
c dorsiflexes foot true
d is innervated by the deep peroneal nerve true
(Greenberg 20. Peripheral Nerves no. 18)
268. True or False. The brachial plexus is formed by the dorsal rami of C5-T1 . false (It is
formed by the ventral rami of C 5-Tl . The dorsal rami innervate the paraspinal muscles.)
(Greenberg 20. Peripheral Nerves no. 19)
269.

Draw a diagram of the brachial plexus.

Fig. 20.1
(Greenberg 20. Peripheral Nerves no. 20)
270.

On your diagram of the brachial plexus, label the following:


(1) roots C4-Tl ; (2) organization RTDCN (roots, trunks, divisions,
chords, nerves); (3) names of trunks-SMI (superior, middle,
inferior); (4)add names of cords-LMP (lateral, medial, posterior)

(Greenberg 20. Peripheral Nerves no. 21)

271. Add the nerves to the basic outline of the brachial plexus nerves: 16. Hint: Donald
says somewhat loudly Mickey Mouse you are right to so sincerely love Minnie Mouse
madly.

Fig. 20.3
(Greenberg 20. Peripheral Nerves no. 22)

272.

Draw the complete brachial plexus:

Fig. 20.4
(Greenberg 20. Peripheral Nerves no. 23)
273.

Draw the left brachial plexus -question 20, outline

Fig. 20.5

(Greenberg 20. Peripheral Nerves no. 24)


274.

Draw the left brachial plexus and add details requested in questions 21 through 23

Fig. 20.6
(Greenberg 20. Peripheral Nerves no. 25)
275. Complete the following about the
brachial plexus:
a Name the roots (6}. C4, C5, C6, C7, C8, Tl
b Name the segments (5}. Hint: Run to do Cindy's needs. roots. trunks, divisions,
chords, nerves
c Name the nerves (1 6}. Hint: Donald says somewhat loudly Mickey Mouse you are
right to so sincerely love Minnie Mouse madly.
dorsal scapular
suprascapular
subclavius
lateral pectoral
musculocutaneous
median
ulnar
axillary
radial
thoracodorsal
subscapular upper
subscapular lower
long thoracic
medial pectoral
medial brachial cutaneous
medial antebrachial
cutaneous
d Name the trunks (3} superior, middle, inferior,
e Name the cords (3} lateral, medial, posterior
(Greenberg 20. Peripheral Nerves no. 26)

276. Trace, using the brachial plexus diagram, the theoretically possible root contribution
to each nerve and then compare with the actual root contribution in each nerve.
a nerve dorsal scapular, theoretical C4,5 , actual, C4,5,
b nerve suprascapular , theoretical C4,5,6 , actual C 4,5,6
c nerve subclavius, theoretical C 6, actual C 6
d nerve lateral pectoral, theoretical C 4,5,6,7, actual C 4,5,6,7
e nerve musculocutaneous, theoretical C 5,6,7, actual C 5,6,7,
f nerve median, theoretical C 5,6,7 T 1, actual C 5,6,7 T 1
g nerve ulnar, theoretical C 8, T1, actual C 7,8, T1
h nerve axillary, theoretical C4,5,6,7,8, T1 , actual C4,5,6,7,8, T1
(Greenberg 20. Peripheral Nerves no. 27)
277. List the brachial plexus nerves (except for median ulnar and radial), the muscles
they serve, the roots that are inthat nerve, and the action of the muscles:
a nerve dorsal scapular, muscle levator scapulae, root C3,4;5 , action, elevate scapulae
b nerve dorsal scapular, muscle rhomboids, root C 4,5 action, adduct and elevate scapula,
c nerve suprascapular, muscle supraspinatus, root C 4,5,6 , action, adduct arm 0- 90
degrees
d nerve supraspinatus , muscle infraspinatus,, root C 5,6, action rotate arm out
e nerve musculocutaneous , muscle biceps brachii, root C 5,6, action lex and supinate
forearm,
f nerve musculocutaneous , muscle coracobrachialis, root C 5,6,7, action flex and adduct
forearm,
g nerve musculocutaneous , muscle brachialis, root C 5,6, action flex forearm
h nerve axillary , muscle deltoid, root C 5,6 , action abduct arm > 90 degrees
i nerve subscapularis , muscle teres major, root C5,6,7, action adducts arm
j nerve thoracodorsal , muscle latissimus dorsi, root C5,6,7,8 , action adduct arm ladder,
cough,
k nerve axillary , muscle teres minor , root C4,5 , action rotation lateral
l nerve long thoracic , muscle serratus anterior , root C5,6,7 , action forward shoulder
thrust
(Greenberg 20. Peripheral Nerves no. 28)
278. Considering the brachial plexus and radial nerve, list the branches of the radial
nerve cascade in proper sequence and the function of the muscles. Hint: rest in peace,
retbes in peeeeeae
radial
extensor
triceps
brachioradialis
extensor carpi radialis
supinator
i
n posterior interosseres nerve
p
extensor carpi ulnaris
extensor digitorum communis

extensor digit minimi


extensor pollicis brevis
extensor pollicis longus
abductor pollicis longus
extensor indicis
(Greenberg 20. Peripheral Nerves no. 29)
279. True or False. The radial nerve is formed by
a CS-Tl false
b CS-C 8 true
c C6-Tl false
d CS-C7 false
(Greenberg 20. Peripheral Nerves no. 30)
280. True or False. Regarding the radial nerve, it
a is formed by C 5-C 8 true
b innervates triceps true
c innervates supinator true
d innervates brachioradialis true
e continues into forearm as posterior interosseus nervetrue
(Greenberg 20. Peripheral Nerves no. 31)
281. What is innervated by the axillary nerve? Hint: tm and d
a teres minor
b deltoid
(Greenberg 20. Peripheral Nerves no. 32)
282. Regarding the brachial plexus and median nerve, list the 11 branches of the median
nerve cascade in proper sequence. Hint: pf pf3 pf aol
pronator teres
flexor carpi radialis
palmaris longus
flexor digitorum superficialis
flexor digitorum profundus
flexor pollicis longus
pronator quadratus
flexor pollicis brevis
abductor pollicis brevis
opponens pollicis
lumbricales 1 and 2
(Greenberg 20. Peripheral Nerves no. 3)
283. Regarding the brachial plexus and median nerve, list the 11 branches of the median
nerve cascade and the function of the muscles:
a pronator teres
i. function: forearm pronator

b
i. f flexor carpi radialis
ii. function: radial flexion of hand
c
i. palmaris longus
ii. function: hand flexion
d
i. flexor digitorum superficialis
ii. function: flex middle phalanx, fingers 2 , fingers to 5
e
i. flexor digitorum profundus
ii. function: flex distal phalanx, fingers 2 fingers to 3
f
i. flexor pollicis longus
ii. function: flex distal phalanx of thumb
g
i. pronator quadratus
ii. function: pronates forearm
h
i. flexor pollicis brevis
ii. function: flexes procimal phalanx of thumb
i.
i. abductor pollicis brevis
ii.function: abducts thumb metacarpal
j
i. opponens pollicis
ii. function: opposes thumb metacarpal
k
i. 1 and 2 lumbricales
ii. function: extend 2 distal phalanges of fingers 4 and 5
(Greenberg 20. Peripheral Nerves no. 34)
284. Which muscles in the hand are innervated by the median nerve? Hint: loaf
lumbricals 1 and 2
opponens pollicis
abductor pollicis brevis
flexor pollicis brevis
(Greenberg 20. Peripheral Nerves no. 35)
285. Which muscles are served by the anterior interosseous nerve?
flexor digitorum profundus
flexor pollicis longus
pronator quadratus
(Greenberg 20. Peripheral Nerves no. 36)
286. Regarding the brachial plexus and ulnar nerve, list the muscles served by the ulnar
nerve cascade in proper order and the function of the muscles. Hint: "Ffafner I Love
Him"

a. i. flexor carpis ulnaris


ii. function: ulnar flexion of hand
b. i flexor digitorum profundus
ii function flex distal phalanx of fingers 4 and 5
c. i. adductor pollicis
ii function thumb adductor
d i. flexor pollicis brevis
ii. function: flex proximal phalanx of thumb
e
i. interossei
ii. function: dorsal abducts
iii. function: palmar abducts flex proximal phalanges at metacarpalphalangeal
joints
f
i. lumbricates
ii. function: extends two distal phalanges 3 and 4 at interphalangeal joints
g
i hypothenar abductor digiti
ii. function: minimi, flexor digiti minimi , opponens
iii. function: abduction little finger , flex little finger
(Greenberg 20. Peripheral Nerves no. 37)
287. Study Chart
Radial
Ulnar
RETBES in FFAF ILH
PESAE
RETBES in

PEEEEEAE

Median

Radial

PFPF3PFAOL radial
extensor

triceps
brachioradiali
s
exteilsor carpi
radialis

Ulnar

Median

flexor carpi
ulnaris
flexor
digitorum
profundus
adductor
pollicis
flexor
pollicis
brevis
interossei

pronator
teres
flexor carpi
radialis

supinator

lumbricales

hypothenar

almaris
longus
flexor
digitorum
superficialis
flexor
digitorum
profundus
flexor
pollicis
longus
(AIN)
flexor
pollicis
brevis
abductor
pollicis
brevis

n
p

(PIN)

opponens
pollicis
opponens
pollicis

extensor carpi
ulnaris
extensor
digitorum
extensor digiti
minimi
extensor
pollicis brevis
extensor
pollicis
longus
abductor
pollicis
longus
extensor
indicis
(Greenberg 20. Peripheral Nerves no. 38)
288. Which muscles in the arm are innervated by the ulnar nerve? none
(Greenberg 20. Peripheral Nerves no. 39)
289. Regarding the following additional (2) nerves of the brachial plexus, number the
roots and name the muscles and their actions:
nerve musculocutaneous, roots C 5,6,7, muscles (1) biceps, (2) coracobrachialis, (3)
brachialis, action (1) flex forearm and supinates,(2) flex forearm and adducts, (3) flex
forearm
nerve axillary , roots C4,5,6 , muscles (1) deltoid, (2) teres minor action (1) abduct arm
30 to 90 degrees, (2) lateral arm rotation
(Greenberg 20. Peripheral Nerves no. 40)
290. Peripheral neuropathy-list the etiology: Hint: dang the rapist
diabetes
alcohol
nutritional, B12
Guillain-Barre
traumatic
hereditary
entrapment
renal, radiation
amyloid

porphyria, paraneoplastic
infectious, Hanson's
sarcoidosis
toxins, heavy metals
(Greenberg 20. Peripheral Nerves no. 41)
291. Regarding peripheral neuropathy:
The most common peripheral neuropathy that is an inherited disorder is Charcot-MarieTooth syndrome
The percent of patients with diabetes mel litus who develop diabetic neuropathy is 50%
(Greenberg 20. Peripheral Nerves no. 42)
292. Which syndrome is associated with pure sensory neuropathy? paraneoplastic
syndrome (also seen with pyridoxine therapy)
(Greenberg 20. Peripheral Nerves no. 43)
293. True or False. Alcohol neuropathy includes:
a motor neuropathy . false
b sensory neuropathy . true
c absent Achilles reflex . true
d intense pain. False
(Greenberg 20. Peripheral Nerves no. 44)
294. True or False. The most important study in the diagnosis of lumbosacral plexus
neuropathy is:
a magnetic resonance imaging (MRI) . false
b computed tomography (CT) . false
c electromyography (EMG) . true (EMG in lumbosacral neuropathy-rule out diabetic
neuropathy!)
d erythrocyte sedimentation rate (ESR). False
(Greenberg 20. Peripheral Nerves no. 45)
295. EMG in lumbosacral neuropathy shows what in regards to:
a fibrillation potententials increased
b motor unit potentials in number decreased
c motor unit potentials in amplitude increased
d motor unit potentials in duration increased
e motor unit potentials that are polyphasic
f have changes involving at least 2 segments
g spare diagnostic the paraspinal muscles is highly
(Greenberg 20. Peripheral Nerves no. 46)
296. True or False. Femoral neuropathy includes:
a weakness of quadriceps and il iopsoas . true
b patellar reflex-reduced .true
c femoral stretch-positive .true

d sensation over lateral calf reduced. False (Femoral neuropathy includes


over anterior thigh and medial calf.)
(Greenberg 20. Peripheral Nerves no. 47)

sensation

297. Regarding femoral neuropathy:


a Name the muscle responsible for
knee extension quadriceps femoris
hip flexion iliopsoas
b To distinguish L4 radiculopathy from femoral neuropathy. L4 radiculopathy would
not involve the il iopsoas
c Femoral neuropathy is caused by:
i. diabetes
ii compression
(Greenberg 20. Peripheral Nerves no. 48)
298. True or False. The most frequent cause femoral neuropathy is:
a intraabdominal tumor . false
b retroperitoneal hematoma . false
c diabetes . true (Diabetes is the most frequent cause. All optionscan cause femoral
neuropathy.)
d entrapment due to inquinal hernia . false
e trauma. False
(Greenberg 20. Peripheral Nerves no. 49)
299. True or False. Regarding AIDS neuropathy:
a It usually presents as proximal symmetric polyneuropathy. false {It is a distal
symmetric polyneuropathy.)
b just HIV+ patients do not develop it. true
c It never includes sensory elements. false (usually includes numbness and tingling)
d It has infectious etiology. true
e It may be caused by lymphomatous invasion of the meninges or nerves.true
(Greenberg 20. Peripheral Nerves no. 50)
300. Describe the anatomy of the peripheral nerve:
a Which connective tissue membrane surrounds individual axons? endoneurium
surrounds individual axons
b Which surrounds groups of axons (i. e . fascicles)? perineurium bundles axons
(covered by endoneurium) into fascicles
c Which surrounds groups of fascicles (i.e., nerves)? epineurium groups fascicles
(covered by perineurium) into nerve trunk
(Greenberg 20. Peripheral Nerves no. 51)
301. Regarding injury and regeneration of nerve:
a The regeneration rate 1 mm/day (i. e., 1 inch per month)
b Sunderland system
first degree anatomy preserved; conduction block, compression, or ischemia

second degree axon injured; endo-, peri-, connective tissue is epineurium intact,
(endoneurium provided tube for regeneration) _
third degree axon and endoneurium disrupted (grossly normal appearance, recovery
related to extent of intrafascicular fibrosis)
(Greenberg 20. Peripheral Nerves no. 52)
302. Complete the following about the peripheral neuropathies:
a fourth degree axon endoperi interruption of all elements but epineurium is intact,
nerve is indurated and enlarged
b fifth degree axon endoperi and epineurium is completely transected
c sixth degree mixed first through fourth through degree injuries completely
(Greenberg 20. Peripheral Nerves no. 53)
303. Complete the following about the peripheral neuropathies:
a Nerve regeneration occurs at the rate of 1 mm/day.
b Nerve regeneration occurs at the rate of 1 inch/fmonth.
c Describe injury classification of peripheranerves and regeneration prognosis.
axon compressed two classifications: Seddon and Sunderland first degree = Seddon
neuropraxia; conduction block from compression or ischemia; anatomy preserved
axon injured second degree = Seddon axonotmesis; injury to axon with Wallerian
degeneration; endoneurium/perineurium/ epineurium intact; endoneurium provides
"tube" to optimize successful reinnervation of target muscle
axon and endoneurium disrupted third degree = axon and endoneurium disrupted;
recovery inversely related to interfascicular fibrosis; gross normal appearance
axon, endoneurium and perineurium disrupted fourth degree = interruption axon.
endoneurium, perineurium; gross reveals indurated enlarged nerve
axon endo-, peri-, and epineurium disrupted fifth degree = Seddon neurotmesis;
complete transection of axon, endo-, peri-, epineurium
(Greenberg 20. Peripheral Nerves no. 54)
304. What are etiologies of brachial plexus injuries? Hint: cpt
compression
penetration
traction
(Greenberg 20. Peripheral Nerves no. 55)
305. Traction (stretch) injuries of the brachial plexus selectively:
a spare the
i.
medial cord
ii.
median nerve
b injure the
i.
posterior cord
ii.
lateral cord
(Greenberg 20. Peripheral Nerves no. 56)

306. Complete the following about the peripheral neuropathies:


a What nerve injury cannot be repaired? proximal to dorsal root ganglion (i. e.,
preganglionic)
b What is the evidence for such an injury? Hint: prEHms
pain , rhomboids , EMG , Horner , meningocele , scapula
(Greenberg 20. Peripheral Nerves no. 57)
307. List the characteristics of Erb and Klumpke brachial plexus injury:
extended
rotated
pronated
claw
lower roots C 8 T1
ulnar type claw plus
median type claw
palsy
(Greenberg 20. Peripheral Nerves no. 58)
308. Describe upper and lower brachial plexus injury:
a upper brachial plexus injury
Duchenne-Erb palsy
upper plexus CS, C6
forceful separation humeral head from shoulder
commonly dystocia or motorcycle crash
internally rotated arm with extended elbow
bellhop's tip, hand not affected
b lower brachial plexus injury
Klumpke palsy
lower plexus C 8, Tl
sudden pull of abducted arm in
fall or Pancoast tumor syndrome
claw hand with weakness/
wasting of small hand muscles
simian hand
(Greenberg 20. Peripheral Nerves no. 59)
309. Brachial plexus birth injuries
a most common is upper
i.
consisting of CS -C 6 50 % and
ii.
CS -C 6-7 25 %
iii.
lower C8-Tl 2 %
b combined is 20 %
c bilateral 4 %
d spontaneous recovery is 90 %
(Greenberg 20. Peripheral Nerves no. 60)

310. Characterize upper brachial plexus injury-Erb palsy:


a roots involved C 5 (ABCDE) fifth letter of alphabet Erb palsy mainly C5 and also
C6, C7
b position of upper extremity: Hint: erp
i.
extended
ii.
rotated
iii.
pronated
looks like bellhop's tip position
c . Weak muscles and their roots:
deltoid, roots C 5,6
biceps, roots C 5,6
rhomboids , roots C 4,5
brachioradialis ,roots C 5,6
supraspinatus , roots C4, CS, C6
infraspinatus, roots CS, C6
d mechanism shoulder separation
e from:
i.
birth injuries
ii.
motorcycle accidents
(Greenberg 20. Peripheral Nerves no. 61)
311. Characterize lower brachial plexus injury
-Kiumpke palsy
a roots involved C7, C8, Tl
b position of upper extremity: Hint: klump
claw hand (Simian hand)
ulnar claw
plus median claw
paralysis
c weak muscles
upper extremity small muscles of hand face Horner if Tl involved
d. mechanism: traction on arm abducted
e from:
falls
birth
Pancoast tumors
(Greenberg 20. Peripheral Nerves no. 62)
312. Regarding birth injury of brachial plexus:
a incidence is 0.3 - 2/1000 births
b
upper 50% CS, C6
upper plus C7 25% CS, C6, C7
c mixed 20%
d lower 2% C7, Tl

e bilateral 4%
(Greenberg 20. Peripheral Nerves no. 63)
313. True or False. The following are indications for early surgical exploration of the
brachial plexus:
any injury needs repair . false (most injuries maximal deficit at onset then improve)
progressive deficit . true (progressive deficit likely vascular injury, explore immediately)
clean sharp injury . true (clean, sharp, fresh lacerating injuries -+ explore acutely and
repair end-tcrend tension-free within 72 hours)
gunshot wound (GSW) to brachial plexus. false (surgery is of little benefit)
(Greenberg 20. Peripheral Nerves no. 64)
314. List medical etiologies of entrapment neuropathies:
arthritis rheumatoid
acromegaly
amyloidosis
polymyalgia rheumatica
carcinomatosis
diabetes
gout
hypothyroidism
(Greenberg 20. Peripheral Nerves no. 65)
315. Name two most common syndromes of median nerve entrapment:
carpal tunnel syndrome
pronator teres syndrome
(Greenberg 19. Stereotactic Surgery no. 66)
316. Describe carpal tunnel syndrome (CTS) anatomy:
a The median nerve passes under the transverse carpal ligament
b The motor branch either goes:
under or
pierces the ligament
c and serves the loaf muscles,
d which are:
limbricales 1 and 2
opponens pollicis
abductor pollicis
flexor pollicis brevis
(Greenberg 20. Peripheral Nerves no. 67)
317. Concerning carpal tunnel syndrome:
a The transverse carpal ligament extends how far beyond the distal wrist crease? 3 cm
b What is the name of the sensory nerve? palmar cutaneous branch
c It arises 5,5 cm em proximal to the wrist.

d It passes above the transverse carpal ligament.


e and serves the thenar eminence sensation.
(Greenberg 20. Peripheral Nerves no. 68)
318. Complete the following about the median nerve:
a Describe the sensory distribution of the median nerve:
thumb: palmar aspect
fingers: index, middle, and half of ring
thenar eminence and adjacent
radial palm
b crosses above transverse carpal ligament
(Greenberg 20. Peripheral Nerves no. 69)
319. Regarding carpal tunnel syndrome describe the epidemiology:
a What is the most common median nerve entrapment neuropathy? carpal tunnel
syndrome
b It is due to compression median nerve
c Where? distal to wrist crease
d age distal to wrist crease
e male/f emale ratio 4 : 1
f bilateral > 50%
g worse in dominant hand
h Phalen sign is performed by forced flexion of the wrist and is positive in 80 %.
(Greenberg 20. Peripheral Nerves no. 70)
320. What is double-crush syndrome?
a involves two sites
cervical radiculopathy
median/ulnar neuropathy
b exacerbated by neck movement
c pathophysiology
postulated that cervical compression
compromises axoplasmic flow
predisposing nerve to distal injury
(Greenberg 20. Peripheral Nerves no. 71)
321. Regarding carpal tunnel syndrome:
What is the most sensitive electrodiagnostic test for carpal tunnel syndrome? sensory
latency nerve conduction velocity (NCV)
Which should be faster, median sensory conduction velocity or ulnar sensory conduction
velocity? median
By how much? 4 m/s faster
(Greenberg 20. Peripheral Nerves no. 72)
322. Regarding carpal tunnel syndrome:
a describe treatment:

splint
steroids
surgery
b Incision should be slightly to the ulnar side of the interthenar crease
c to avoid
palmar cutaneous branch
anomalous recurrent thenar
motor branch
(Greenberg 20. Peripheral Nerves no. 73)
323. Describe main trunk median nerve compression:
a above elbow due to Struther ligament supracondylar to medial epicondyle, mostly
asymptomatic
b at elbow
i.
b a ___ _ bicipital aponeurosis
ii.
p t. ___ _ pronator teres
iii.
s b ___ _ sublimis bridge
c Honeymoon paralysis is due to direct compression
d Benediction hand is due to weakness of flexor digitorum profundus I and II.
(Greenberg 20. Peripheral Nerves no. 74)
324. Characterize pronator teres syndrome (PTS):
a compresses the nerve median
b where it dives between the two heads of the pronator teres
c Symptoms are:
i.
pain in the palm
ii.
weakness in the grip
iii.
paresthesias in the thumb and index fingers
d i. Differs from CTS in that there is no nocturnal pain in pronator teres syndrome
ii. but there is pain in the palm in PTS
iii. because the median palmar cutaneous branch is compressed in PTS and spared
in carpal tunnel syndrome
(Greenberg 20. Peripheral Nerves no. 75)
325. Describe pronator teres syndrome:
a.
i.
caused by repeated pronation
ii.
with a tight fist
b.
i.
due to nerve trapped where it dives between
ii.
two heads of the pronator teres
c .symptoms of:
ache
pain in palm
weak grip
d Distinguished from carpal tunnel syndrome by:
no nocturnal exacerbation

pain in palm
(Greenberg 20. Peripheral Nerves no. 76)
326. What are the key features of anterior interosseous neuropathy?
a.
flexion
distal phalanges
thumb
index finger
b.due to
weakness of the flexor digitorum profundus and the
flexor pollicus longus
c.no loss of sensation (anterior interosseous is pure motor)
d. patient can't make "OK" sign
e treatment:
no identifiable causeexpectant; management 8 to 12 weeks
if no improvement or if progression proceed with
surgical exploration
(Greenberg 20. Peripheral Nerves no. 77)
327.Regarding the anterior interosseous nerve:
a If injured a person can't do what with the thumb and index finger ? make an "0"
b because there is weakness of the:
i. flexor digitorum profundus and
ii. flexor pollicis longus
c Is part of what nerve? median
d Syndrome may be caused by constricting ligament.
e Is there any sensory loss? no sensory loss
(Greenberg 20. Peripheral Nerves no. 78)
328.Regarding the ulnar nerve:
a Name the roots: ulnar components C7, C8, T1
b Motor findings of entrapment? Hint: abcWF
interossei wasting; atrophy,
particularly thumb web space
benediction hand
claw deformity
Wartenberg sign: abducted
little finger
Froment thumb sign
c
i. pain and tingling in little finger
ii. and ulnar half ring finger
(Greenberg 20. Peripheral Nerves no. 79)
329.Ulnar nerve entrapment results in:

a What occurs to interossei? atrophy


b Little finger weak on
i. adduction is called
ii. Wartenberg sign (l ittle finger held in abduction)
c Holding a piece of paper requires modification because of a weak
i. adductor pollicis and is called
ii. Froment prehensile thumb sign
d Waving goodbye demonstrates a claw deformity of the hand
i. also known as main in griffe
ii. also known as benediction hand
e What other nerve injury can produce:
i. benediction hand? median
ii. upon what attempted action? making a fist
(Greenberg 20. Peripheral Nerves no. 80)
330.Describe Wartenberg sign:
a It affects the little finger
b What occurs to the ? little finger
c It rests in abduction
d due to weakness of the third palmar interosseous muscle
e Which nerve is involved? Ulnar
(Greenberg 20. Peripheral Nerves no. 81)
331.Describe Froment sign:
a Test by having the patient ___ _ grasp a piece of paper
b using his thumb and index fingers
c If the nerve is weak what happens? ulnar
d Thumb bends backward (i. e., flexing the distal phalanx or extending proximal
phalanx of the thumb)
e Because ulnar innervated adductor pollicis is weak
f Therefore the body substitutes for it the stronger flexor pollicis longus
g which is innervated by the anterior inerosseous nerve, of the median nerve.
(Greenberg 20. Peripheral Nerves no. 82)
332.Describe ulnar nerve entrapment:
a Injury above elbow due to:
i. injury to medial cord
ii. kinking at the a arcade of Struthers aponeurotic band
b Entrapment at the elbow
AKA as "tardy ulnar palsy" (delayed presentation-initial case 12 years > from injury to
elbow elbow dislocation/lateral condyle fracture; nerve is superficial, fixed and crosses
joint)
NCV is less than 48 m/s
or a difference between the 2 slides of greater than 10 m/s\
c Entrapment in the forearm
d Entrapment in the wrist/hand
(Greenberg 20. Peripheral Nerves no. 83)

333.What are surgical treatment options for ulnar compression at the elbow?
a de without simple nerve decompression without transposition
b de nerve decompression with transposition
c medial epicondylectomy
d Results in %:
i. excellent 60 %
ii. fair 25 %
iii. poor 15 %
e True or False. What responds better ?
i. pain and sensory loss . true
ii. weakness and atrophy . false
(Greenberg 20. Peripheral Nerves no. 84)
334. Entrapment in the forearm-<:ubital tunnel syndrome:
a Involves which nerve? ulnar
b Due to which muscle? flexor carpi ulnaris (Just distal to the elbow, the ulnar nerve
passes from the groove between the)
c The mechanism is compression between the medial epicondyle and the olecranon
process to enter the two heads of the flexor carpi ulnaris under the fascial band
connecting the two heads (the cubital tunnel)
d results in: atrophy of the interrossei, Wartenberg sign, Froment prehensile thumb
sign, claw deformity of the hand (main en griffe)
(Greenberg 20. Peripheral Nerves no. 85)
335. Characteristics of the cubital tunnel syndrome are: Hint: ccubittal
claw deformity
(flexor) carpi ulnaris
ulnar nerve
band is tight
interossei atrophied
thumb sign Froment prehensile
two heads of flexor carpi ulnaris
atrophy of interossei
elbow epicondyle
(Greenberg 20. Peripheral Nerves no. 86)
336. Describe the borders of the Guyon canal:
a roof:
palmar fascia ,
palmar brevis muscle
b floor
flexor retinaculum of the palm
pisohamate ligament
c Below the floor is the transverse carpal ligament
d It contains only the ulnar (At the middle of the canal the nerve divides into deep and
superficial branch. Superficial branch is mostly sensory [except for the branch to palmar

brevis) and supplies hypothenar eminence and ulnar half of ring finger. The deep
[muscular) branch innervates hypothenar muscles, lumbricals 3, 4, and interossei. nerve
and artery.
(Greenberg 20. Peripheral Nerves no. 87)
337. Describe types of ulnar nerve lesions in Guyon canal type-location of compressionweakness-sensory deficit.
a type I
location of compression just proximal to or within Guyon canal,
weakness all intrinsic muscles innervated by ulnar nerve ,
sensory deficit palmar ulnar distribution (palmar ulnar distribution: the
hypothenareminence and ulnar half of ring finger both on the palmar surface only)
b type II
location of compression along deep branch
weakness muscles innervated by deep branch (depending on location may spare
hypothenar muscles)
sensory deficit none
c type Ill
location of compression distal end of Guyon canal
weakness none
sensory deficit palmar ulnar distribution (the hypothenar eminence and ulnar half of ring
finger both on the palmar surface only)
(Greenberg 20. Peripheral Nerves no. 88)
338. Regarding radial nerve injuries:
a. Sensation loss in the web space of the thumb indicates injury in the hand
b. Pain at the lateral epicondyle indicates compression of the supinator tunnel at the
elbow
c.
i Finger drop indicates injury to the PIN
ii. resulting from entrapment at the arcade of Frohse
d.
i.
Wrist drop indicates injury to mid-upper arm
ii.
where the nerve is in the spiral groove along deep branch
e. Triceps plus all distal muscle weakness axilla indicates injury at the axilla
f. above plus weakness of the deltoid and posterior cord latissimus dorsi indicates
injury to the posterior cord
g. above plus winging of the scapula on the roots forward shoulder thrust indicates
injury to the roots.
(Greenberg 20. Peripheral Nerves no. 89)
339. Differentiate radial nerve injury from brachial plexus posterior cord injury.
a Check the function of the and deltoid
b latissimus dorsi radial nerve arises from posterior divisions of the three trunksof the
brachial plexus to form the posterior cord. Sparing of deltoid (axillary) and latissimus
dorsi (thoracodorsal) localizes injury to radial nerve and not the more proximal portion
of the posterior cord. muscles.

(Greenberg 20. Peripheral Nerves no. 90)


340. Differentiate axilla and mid-upper arm radial nerve compression:
a Check the function of the triceps muscle.
b Wrist drop plus weak triceps implicates injury at axilla: crutch misuse, weak triceps
and distal, radial innervated muscles
c Wrist drop but normal triceps implicates injury at mid-upper arm; sites: spiral
groove, intermuscular septum; improper arm positioning with; intoxication "Saturday
night palsy"; iatrogenic surgical positioning; callus old humeral fracture; wrist dropnormal triceps; DDX (lead poisoning)
(Greenberg 20. Peripheral Nerves no. 91)
341. Describe mid-upper or forearm radial nerve compression:
a Radial nerve compression mid-upper arm produces
weakness (wrist drop)
numbness
PIN and superficial (sensory) radial nerve (finger drop)
b Injury to the posterior interosseous nerve (PIN) produces
weakness
numbness
PIN (motor) and not the superficial radial nerve (sensory)
c Injury at the supinator tunnel produces
pain but no
weakness and no
numbness
(Greenberg 20. Peripheral Nerves no. 92)
342. Complete the following about the peripheral neuropathies:
a PIN refers to the posterior interosseous nerve
b a continuation of the nerve, radial
c which serves the
extensors of the fingers
abductor pollicis longus
d deltoid
action abduct arm from 30 - 90 degrees,
roots C 5,6,
nerve thoracodorsal,
e latissimus dors
i.
action, __ adduct arm, cough, ladder climb
ii.
roots, __ CS, C6, C7, C8
iii.
nerve, _ thoracodorsal
(Greenberg 20. Peripheral Nerves no. 93)
343. Concerning the radial nerve and wrist weakness:
a Failure of wrist extension (wrist drop) indicates proximal radial nerve injury.

b Failure of finger extension (finger drop) PIN indicates injury.


(Greenberg 20. Peripheral Nerves no. 94)
344. Describe forearm/hand radial nerve compression management:
a posterior interosseous syndrome Surgical exploration if no improvement after 4 to 8
weeks expectant management. Lyse constrictions and arcade of Frohse.
b supinator tunnel syndrome Responds to nerve decompression. Lyse constrictions and
extensor carpi radialis brevis.
c hand injury
i.
Clinically you find causes small area of sensory loss dorsal
ii.
at the web space of thumb of
iii.
often caused by handcuffs
iv. symptoms are mild so no surgery is needed
(Greenberg 20. Peripheral Nerves no. 95)
345. Describe the suprascapular nerve:
a formed from roots C 5, 6
b entrapped at transverse scapular ligament (T5L) (History: antecedent frozen
shoulder or trauma)
c sensory symptoms Referred, poorly localized shoulder pain. Nerve innervates joint
capsule, no cutaneous representation.
d motor symptoms:
i.
atrophy of infraspinatus and supraspinatus
ii.
weakness of a supraspinatus upper from 0 to 30 degrees
iii.
weak backhand tennis shot
e Is EMG helpful? yes, to distinguish from rotation cuff injury
f Treatment surgery; if fails to improve cut T5L
g Differentiate from C 5 cervical radiculopathy and upper brachial plexus
lesion by testing rhomboid and deltoid (will show weakness in C 5 radiculopathy
(Greenberg 20. Peripheral Nerves no. 96)
346. Define meralgia paresthetica:
a hyperpathia located at the lateral upper thigh (burning pain with hyperpathia)
b entrapment of the lateral femoral cutaneous nerve
c True or False. It contains motor and sensory false (pure sensory L2, L3) fibers.
(Greenberg 20. Peripheral Nerves no. 97)
347. Complete the following about the peripheral neuropathies:
a Which is the most common nerve to develop acute compression palsy? the common
peroneal nerve
b At what location? fibular head
c It results in impairment of:
i.
motor function: foot drop
ii.
sensory loss: dorsum of foot
(Greenberg 20. Peripheral Nerves no. 98)
348.

Matching. Match the following Nerve Also known as:


(1) musculocutaneous

(2) medial popliteal


(3) lateral popliteal
(4) anterior tibial
a. tibial L4-5, 52-3 (2)
b. common peroneal L4-5, 51 (3)
c. deep peroneal L4-5, 51 (4)
d. ficial peroneal LS, 51 (1)
(Greenberg 20. Peripheral Nerves no. 99)
349. Match the nerve with the function it serves:
Nerve Functions:
(1) plantar flexors and inversion
(2) origin of deep and superficial peoneal
(3) dorsiflexors superation toe extensors
(4) plantar flexors and eversion
1. tibial (1)
2. common peroneal (2)
3. deep peroneal (3)
4. superficial peroneal (4)
(Greenberg 20. Peripheral Nerves no. 100)
350. Match the following nerve and its area of isolated sensory loss:
Nerve Area of isolated sensory loss:
(1) lateral aspect of calf and dorsum of foot
(2) space between great and second top
1. deep peroneal (2)
2. superficial peroneal (1)
(Greenberg 20. Peripheral Nerves no. 101)
351. Matching. Match the nerve with its characteristics.
Characteristic:
(1) passes behind the fibular head; (2) is the common nerve to develop
acute compression palsy; (3) serves the foot extensors; (4) serves
the foot evertors; (5) space between great toe and second toe; (6)
dorsum of foot
Nerve:
a common peroneal (1,2)
b deep peroneal (3,5)
c superficial peroneal (4,6)
(Greenberg 20. Peripheral Nerves no. 102)
352. True or False. Loss of pinprick sensation to the web space between the great toe and
first toe can occur with:
a superficial peroneal nerve compression false (sensory loss lateral leg and dorsum of
foot)
b deep peroneal compression .true
c S 1 nerve root compression . false (sensory loss to latera foot and little toe)
d none of the above. False

(Greenberg 20. Peripheral Nerves no. 103)


353. True or False. Entrapment of the common peroneal nerve at the fibular head may
result in:
a weak soleus muscle false (innervated by the tibial nerve)
b foot drop . true
c weak biceps femoris muscle . false (biceps femoris innervated by sciatic proximal to
take off of common peroneal)
d sensory impairment in the lateral calf and dorsum of foot. true (foot drop and
sensory impairment in lateral calf and dorsum foot
(Greenberg 20. Peripheral Nerves no. 104)
354. True or False: A foot drop may result from:
a parasagittal meningioma . true
b deep peroneal nerve palsy . true
c LS radiculopathy (occasionally L4) . true (LS is more commonly the cause of foot
drop.)
d superficial peroneal nerve palsy . false (There is weakness of foot eversion but not
foot drop.)
e common peroneal nerve palsy. True
(Greenberg 20. Peripheral Nerves no. 105)
355. True or False. Peroneal nerve palsy may result from:
a diabetes mellitus . true
b clipping injury in a football player . true
c venous thrombosis . true
d leprosy (Hansen disease). True
(Greenberg 20. Peripheral Nerves no. 106)
356. True or False. The posterior tibial nerve may be:
a found in the tarsal tunnel . true
b found posterior and inferior to the medialmalleolus . true
c trapped at the retinacular ligament .true
d classically responsible for nocturnal pain and paresthesia at the heel. false (Heel is
spared. Paresthesias are in the toes and sole of the foot.)
(Greenberg 20. Peripheral Nerves no. 107)
357. Matching. Match the following nerves with their functions and alternate
names: Function and alternate name:
(1) also known as musculocutaneous; (2) also known as medial
popliteal;(3) also known as lateral popliteal; (4)also known as
anterior tibial; (5)serves plantar flexors of foot plus inversion; (6 )
origin of deep plus sup P; (7) foot dorsiflexors supination and toe
extensors; (8) foot plantar flex and eversion; (9)space between
great and second toe;(10) lateral aspect of the calf and dorsum of
foot
Nerve:

a tibial L4, 5, S2, S3 (2,5)


b common peroneal (3,6)
c deep peroneal L4, 5, 51 (4,7,9)
d superficial peroneal L 5, 51 (1,8,10)
(Greenberg 20. Peripheral Nerves no. 108)
358. True or False. Clinical presentation of the thoracic outlet syndrome may include:
a pallor and ischemia of hand and fingers . true
b arm swelling and edema . true
c brachial plexus lower trunk dysfunction . true
d brachial plexus medial cord dysfunction. True
(Greenberg 20. Peripheral Nerves no. 109)
359. True or False. Regarding the thoracic outlet syndrome, conservative treatment may
be as effective as the surgical treatment. True
(Greenberg 20. Peripheral Nerves no. 110)
DIRECTIONS: Each item below contains a question or incomplete statement followed
by suggested responses. Select the one best response to each question.
Items 360363
A 32-year-old man living along the coast of Massachusetts presents with an acutely
evolving left facial weakness. Although he has no facial pain or numbness, he does
complain of diffuse headache. He has no history of diabetes mellitus or other systemic
illnesses, but does report newly appearing joint pains and a transient rash on his right leg
that cleared spontaneously more than 1 month prior to the appearance of the facial weakness. On examination, he has mild neck stiffness and pain on hip flexion of the extended
leg.
360. This man is at highest risk for which of the following causes of a unilateral facial
weakness?
a. HIV-associated neuropathy
b. Lyme neuropathy
c. Diphtheritic polyneuropathy
d. Tuberculous meningitis
e. Schwannoma
Jawab:B
361. The physician concludes that the patient has tuberculous meningitis and treats him
with isoniazid and rifampin. To avoid additional signs of neuropathy, which of the
following agents should be administered along with these antibiotics?
a. Ceftriaxone
b. Thiamine
c. Erythromycin
d. Vitamin B12
e. Pyridoxine
Jawab:E
362. Because the facial weakness persists despite antituberculous treatment, the physician
decides to administer high-dose steroids. Within 1 week of the introduction of prednisone,
the patient develops pain radiating down the back of his right leg and difficulty

dorsiflexing the right foot. This new complaint probably represents which of the
following disorders?
a. Borrelia radiculopathy
b. Diabetic mononeuritis multiplex
c. Isoniazid neuropathy
d. Rifampin toxicity
e. Tuberculous radiculopathy
Jawab:A
363. Eventually, the physician receives results of a diagnostic test and switches the patient
to a cephalosporin. The mans facial strength improves, but he notices twitching of the
left corner of his mouth whenever he blinks his eye. This involuntary movement disorder
is probably an indication of which of the following?
a. Sarcoidosis
b. Recurrent meningitis
c. Aberrant nerve regeneration
d. Mononeuritis multiplex
e. Cranial nerve amyotrophic lateral sclerosis (ALS)
Jawab:C
364. A 25-year-old woman is being examined by her physician. The knee jerk is being
tested. The patellar tendon reflex involves sensory fibers of the femoral nerve that
originate in spinal segments
a. S3S4
b. S2S3
c. S1S2
d. L4L5
e. L2L3
Jawab:E
365. A 51-year-old factory worker has noticed progressive weakness over the past year.
Examination and testing reveal a painless largely motor peripheral neuropathy. Of the
following agents, the one most likely to be etiologic in this case is
a. Lead
b. Manganese
c. Thallium
d. Cyanide
e. Mercury
Jawab:A
366.

The most common cause of mononeuropathy multiplex is


a. Diabetes mellitus
b. Temporal arteritis
c. Sarcoidosis
d. Systemic lupus erythematosus
e. Periarteritis nodosa
Jawab:A

367. A very thin elderly woman complains of left-sided neck pain. Her family attempted to
give her a deep intramuscular injection of steroids. She complains acutely of pain

radiating down her arm and develops a wrist-drop. The probable site of injection is the
a. Posterior cord of the brachial plexus
b. Medial cord of the brachial plexus
c. Lateral cord of the brachial plexus
d. T1 spinal root
e. C5 spinal root
Jawab:A
368.

Injuries limited to the upper brachial plexus are most likely with
a. Node dissections in the axilla
b. Pancoast tumor
c. Birth trauma
d. Dislocation of the head of the humerus
e. Aneurysm of the subclavian artery
Jawab:C

369.

The most prominent areas of degeneration with Friedreichs disease are in the
a. Cerebellar cortex
b. Inferior olivary nuclei
c. Anterior horns of the spinal cord
d. Spinocerebellar tracts
e. Spinothalamic tracts
Jawab:D

Items 370373
A 20-year-old ataxic woman with a family history of Friedreichs disease develops
polyuria and excessive thirst over the course of a few weeks. She notices that she
becomes fatigued easily and has intermittently blurred vision.
370. The most likely explanation for her symptoms is
a. Inappropriate antidiuretic hormone
b. Diabetes mellitus
c. Panhypopituitarism
d. Progressive adrenal insufficiency
e. Hypothyroidism
Jawab:B
371. The peripheral neuropathy that would be expected to be seen with this patient
develops in part because of degeneration in
a. Dorsal root ganglia
b. Spinocerebellar tracts
c. Anterior horn cells
d. Clarkes column
e. Posterior columns
Jawab:A
372.

This patients condition has been consistently linked to a defect on


a. Chromosome 21
b. Chromosome 9
c. Chromosome 6
d. The Y chromosome

e. The X chromosome
Jawab:B
373. If this patient has children, at what stage of life would they be expected to become
symptomatic if they inherited Friedreichs ataxia?
a. Neonatal period
b. Juvenile period
c. Early adulthood
d. Middle age
e. Senescence
Jawab:B
Items 374373
A 17-year-old male presents with 10 days of progressive tingling paresthesias of the
hands and feet followed by evolution of weakness of the legs two evenings before
admission. He complains of back pain. He has a history of a diarrheal illness 2 weeks
prior. On examination, he has moderate leg and mild arm weakness, but respiratory
function is normal. There is mild sensory loss in the feet. He is areflexic. Mental status is
normal.
374. Spinal fluid analysis in this case is most likely to show
a. No abnormalities
b. Elevated protein level
c. Elevated white blood cell (WBC) count
d. Elevated pressure
e. Oligoclonal bands
Jawab:B
375.

The most frequent preceding infection before the onset of this syndrome is
a. HIV
b. Cytomegalovirus (CMV)
c. Chlamydia psittaci
d. Mycoplasma pneumoniae
e. Campylobacter jejuni
Jawab:E

376. Over the course of the following week, he has further evolution of weakness
involving muscles of the arms, face, and respiration. He is intubated and placed in the
intensive care unit. Nerve conduction and electromyogram (EMG) studies show
widespread demyelination. Therapy with which of the following may help to speed
recovery?
a. Corticosteroids
b. Cyclophosphamide
c. Plasma exchange
d. Albumin infusions
e. 3,4-diaminopyridine
Jawab:C
DIRECTIONS: Each group of questions below consists of lettered options followed by a
set of numbered items. For each numbered item, select the one lettered option with which
it is most closely associated. Each lettered option may be used once, more than once, or

not at all.
Items 377380
Match each clinical scenario with the most likely diagnosis.
a. Charcot-Marie-Tooth disease
b. Fabrys disease
c. Riley-Day disease (familial dysautonomia)
d. Parsonage-Turner syndrome (brachial plexopathy)
e. Meralgia paresthetica
f. Chronic inflammatory demyelinating polyneuropathy (CIDP)
g. Acute intermittent porphyria
h. Reflex sympathetic dystrophy
i. Leprosy
j. Critical illness neuropathy
377. A 26-year-old woman develops the acute onset of left shoulder pain. Over the
following week, she develops weakness in the proximal left arm and mild sensory loss.
On examination, she has scapular winging and marked weakness of the left deltoid,
biceps, and triceps muscles. The right side is normal, as are her legs. Mild sensory loss in
the upper arm is found. She has lost her biceps and triceps reflexes. Her brother recently
had a similar problem. (SELECT 1 DIAGNOSIS)
Jawab:D
378. A 4-year-old Jewish child has a history of poor sucking at birth, as well as multiple
respiratory infections during childhood. He is of short stature and has not been able to eat
due to progressive vomiting. On examination, strength is normal, but he is hyporeflexic.
There is sensory disassociation, with loss of pain and temperature sensation and
preservation of tactile and vibratory sense. The corneas are ulcerated, pupils do not react,
and he has orthostatic hypotension. (SELECT 1 DIAGNOSIS)
Jawab:C
379. A 56-year-old woman has slowly worsening numbness and paresthesias of the hands
and feet, as well as proximal muscle weakness. Bulbar muscles are normal. An EMG
shows multifocal conduction block, slowing of nerve conduction, and minimal loss of
amplitude of muscle action potentials. Cerebrospinal fluid (CSF) exam shows an
elevation in protein to 260, but no increase in the number of cells. (SELECT 1
DIAGNOSIS)
Jawab:F
380. A 40-year-old police officer is given pain medications after a femoral fracture. One
week later, he presents with confusion, psychosis, abdominal pain, and vomiting. On
exam, he is tachycardic, hypertensive, and febrile. He appears delirious. His arms are
weak, sensation is relatively preserved, and he is areflexic. His wife relates that he had
similar episodes before, when he was in the military. (SELECT 1 DIAGNOSIS)
Jawab:G
Items 381383
381. For each clinical scenario, select the most likely condition.
a. Diabetes mellitus
b. Sarcoidosis
c. Thiamine deficiency

d.
e.
f.
g.
h.
i.
j.

Pyridoxine deficiency
Friedreichs disease
Nitrous oxide poisoning
Gout
Amyloid
Abetalipoproteinemia
Carcinoma

382. A 49-year-old dentist complains of a pins-and-needles sensation in her feet developing


over the course of 3 months. Results of her serum chemistries, blood count, and urinalysis
are all normal, but her hematocrit is at the lower limit of normal. She has a positive
Lhermittes sign (electrical pain down the back on flexion of the neck). EMG and NC
studies reveal slowed conduction in her sensory nerves. There is no family history of similar problems. (SELECT 1 CONDITION)
Jawab:F
383. A 25-year-old woman with a prior history of visual loss in the left eye and a spastic
gait develops impaired pain and temperature perception in her feet. She was diagnosed
with multiple sclerosis (MS) shortly after her visual loss. Her left fundus reveals optic
atrophy, and her facial movements are asymmetric. Chest x-ray reveals large hilar lymph
nodes. Mammogram reveals no apparent carcinoma. (SELECT 1 CONDITION)
Jawab:B
384. A 41-year-old homeless man complains of severe burning in his feet. Vibration,
position, pain, and temperature senses are all impaired in both of his lower extremities up
to the level of the midcalf. He admits to drinking 1 pt of vodka daily. He was operated on
in the past for bleeding from esophageal varices. (SELECT 1 CONDITION)
Jawab:C

Vous aimerez peut-être aussi